Como estudar medicina nos Estados Unidos?

Diego Meille

Como estudar medicina nos EUA

Esse é um dos assuntos mais pedidos aqui no blog. Já escrevi sobre como Estudar medicina em Harvard, mas muitas pessoas tem pedido para que eu escreva de forma mais generalizada, considerando outras universidades também.

Se você já leu o artigo sobre Harvard, partes deste artigo aqui parecerão repetitivas, mas a fim de esclarecer quem não leu o artigo anterior, preciso repetir certos conceitos novamente.

——————–

**ATUALIZAÇÃO**

Antes de começar, eu preciso esclarecer um ponto que é complicado e chato. Medicina nos EUA é um curso extremamente “fechado” (é o único com essas características). O que eu quero dizer com isso? O que eu quero dizer é que se você não tem Green Card ou não é cidadão Americano (nascido no país ou naturalizado), as possibilidades de entrar em medicina nos EUA são muito pequenas. Quando eu escrevi esse artigo há alguns anos eu não sabia disso. Eu já trabalhei com seleção de alunos de faculdade e pós-graduação, mas nunca em medicina, então quando produzi esse artigo, eu tinha os dados que os Americanos sabem sobre a entrada em medicina, mas até que os leitores começassem a me fazer perguntas, eu não tinha a informação de que estrangeiros sem Green Card não são considerados no processo seletivo. Esse processo de seleção começa com o preenchimento de um formulário online da AMCAS (The American Medical College Application Service®). Essa instituição é que controla todos os candidatos que desejam entrar em medicina. Sem documentação como o CPF Americano (Social Security Number) não é possível nem sequer começar a preencher esse formulário e também não é possível entrar em contato diretamente com a faculdade em que você quer estudar. Em sites como o Reddit (onde é comum encontrar entrevistas coletivas com pessoas de destaque) é possível ler entrevistas com diversos profissionais responsáveis pela seleção de alunos em medicina. Em praticamente todas elas sai a pergunta sobre alunos estrangeiros e esses profissionais são taxativos: sem documentação Americana, a aplicação não é processada e consequentemente, não é considerada como válida. A faculdade de medicina não fornece visto de estudante. Esse artigo, portanto, é útil para os Brasileiros já vivendo nos EUA, Brasileiros que nasceram nos EUA, mas cresceram no Brasil, e filhos de cidadãos Americanos que possuem cidadania ou Green Card. Se você é muito jovem, é possível também traçar uma estratégia para que você consiga esse documento, seja casando com um cidadão Americano ou através de visto de trabalho que conduz após muitos anos ao Green Card definitivo e depois cidadania.

——————–

Se você quer fazer medicina, você vai adorar este livro! Um resumo da história do Sec.XX sob a ótica da medicina. É fantástico![/caption]Eu aproveito para alertar os leitores que eu “aportugueso” muitos termos comumente usados no processo de entrada nas faculdades de língua inglesa (não só nos EUA) até para que o aluno lendo este artigo já se familiarize com a “forma de falar”, como o termo “aplicar”, por exemplo, que se refere a “se candidatar” a uma vaga. Nos EUA se diz: “apply to”, o que “aportuguesado” fica “aplicar”. Está incorreto em Português, mas acho útil usar esses termos para familiarizar o leitor quanto à linguagem utilizada.

É preciso, antes de tudo, compreender que medicina nos EUA, assim como todas as outras “profissões” (direito, psicologia, administração, farmácia, veterinária, odontologia, etc.) são cursos que em nossa linguagem Brasileira, é “pós-graduação” (eles chamam de graduação ou graduate school), ou seja, para entrar em medicina nos EUA, é preciso primeiro ter feito uma faculdade, tendo um diploma de bacharel. Isso soa muito estranho para os Brasileiros, que têm acesso a escolas de nível superior direto do ensino médio. O cabra lê isso aqui e fica com um pulga atrás da orelha: mas como assim, para entrar na faculdade eu preciso fazer outra faculdade primeiro? Hummm???

É preciso compreender que no Brasil, o conhecimento que nos EUA é separado para o college (a primeira faculdade, ou a “subgraduação”, undergraduate school) é dado junto com os próprios cursos. Ou seja, no Brasil, medicina dura 6 anos, pois os dois primeiros servem para dar o aluno essa formação básica que ele precisa ter em termos de iniciação científica e “cultural”. Nos EUA, como o aluno precisa ter essa iniciação separadamente, a faculdade de medicina dura apenas 4 anos. Direito, por exemplo, dura apenas 3. Dá pra entender agora que no Brasil os cursos são mais longos pois incluem o que nos EUA é dado separadamente em outra faculdade (college)?

O college nos EUA se foca principalmente em ensinar os alunos a fazer pesquisa e escrever muito bem. Isso é uma grande desvantagem para o aluno Brasileiro, pois no Brasil não se dá ênfase para a escrita, muito menos para a iniciação científica. O aluno no Brasil só vai aprender a fazer pesquisa direito no mestrado e muitos ainda terminam o mestrado sem saber conduzir uma pesquisa científica apropriadamente e escrevem muito mal. Para estudar nos EUA, você precisa saber se expressar de forma escrita muito bem, pois praticamente todas as matérias se baseiam na produção dos “essays” e “papers” ou redação, artigo, que você deve entregar mostrando o que aprendeu na matéria. É um estilo bem jornalístico de ensino, os alunos precisam produzir artigos como os que são publicados nas melhores revistas científicas. Então se você não escreve bem, comece desde já a melhorar suas técnicas de redação e principalmente, aprenda a escrever em inglês impecavelmente! isso, infelizmente, você não vai aprender nas aulinhas de Inglês no Brasil pois os próprios professores, em sua maioria, jamais escreveram um artigo científico na vida.

A escrita e a pesquisa são cobradas do aluno que tenta entrar em escolas de graduação como medicina e direito nos EUA. Esse é o principal motivo pelo qual o college é separado. As escolas de graduação não perdem tempo ensinando os alunos a fazer pequisa científica e escrever direito, eles já devem ter aprendido isso na faculdade inicial que fizeram e se focam apenas em sua área de estudo. Na realidade, o aluno Americano já obtém um bom conhecimento de produção de texto desde a escola primária. Quando eu fiz administração de empresas no Brasil, meu curso incluía metodologia científica, filosofia e introdução à psicologia, só para citar algumas matérias. Nos EUA, a business school (escola de negócios) dá apenas matérias ligadas à administração em si, pois o aluno já deve ter cursado essas matérias primárias anteriormente. Também é objetivo do college, além de iniciar o aluno na pesquisa e na escrita acadêmica, dar um banho de cultura, com leituras clássicas, desde literatura até filosofia, história da arte, história da ciência, e dependendo da área, estudar as áreas científicas básicas muito bem, como ocorre com medicina e a necessidade de ter estudado física, biologia, química, matemática e cálculo de nível avançado (ou seja, não é o que você estudou no ensino médio!).

Espera-se que o aluno que aplica para a grad school, como medicina e direito, seja um indivíduo totalmente formado cultural e cientificamente. Isso acaba sendo uma desvantagem sem tamanho para o aluno Brasileiro, principalmente aquele que não fez o ensino fundamental e médio em escolas de elite – e teria adquirido um pouco dessa iniciação cultural.

Se você (ou seu filho) quer entrar em medicina nos EUA, em qualquer universidade, é preciso estudar muito bem como esse processo funciona e planejar estrategicamente a (chamada) aplicação para a ‘escola de medicina’. É isso o que os alunos nos EUA fazem. Eles já sabem se querem entrar em medicina ou em direito desde muito cedo e vão tecendo um currículo moldado especificamente para “agradar” o comitê de admissão do curso que querem entrar. Ou seja, se você nunca pensou no assunto, e sendo assim, não se preparou para montar sua aplicação do jeito que os comitês de admissão esperam vê-la, suas chances de entrar em medicina nos EUA são… zero!

Outro ponto muito importante a ser mencionado é que a faculdade de medicina especificamente (ao contrário das outras) é extremamente elitista e preconceituosa no que diz respeito à admissão de alunos estrangeiros. Quem banca as universidades de medicina é o contribuinte Americano. Por esse motivo, quem paga imposto não gosta da ideia de estrangeiros estudando nos EUA, se beneficiando do que o dinheiro deles está pagando para depois de formados, arrumarem as malas e voltarem para casa, levando consigo o conhecimento que obtiveram no país e não retribuindo para a sociedade Norte Americana.

Algumas faculdades de medicina NÃO ACEITAM ALUNOS ESTRANGEIROS, de jeito nenhum!

Mas então como vou estudar medicina nos EUA???

Se bateu o desespero, pera lá! Existem soluções!

Repleto de detalhes históricos fascinantes e maravilhas médicas modernas, este importante livro é um envolvente vislumbre das lutas dos médicos e dos impasses que raramente vem à tona.[/caption]A mais óbvia é conseguir um Green Card, que acaba levando à obtenção da cidadania Norte Americana. Não é o objetivo deste artigo explicar como conseguir um Green Card, então não vamos nos aprofundar nisso, apenas citar que essa “É” a melhor solução.

Outra forma, menos garantida, é fazer college nos EUA (recomendado) e ficar no país com visto de estudante. Se a pessoa não conseguir durante os 4 ou 5 anos de faculdade uma forma de conseguir Green Card (por exemplo, se casando com um Americano/a), ela pode argumentar durante a entrevista na faculdade de medicina que tem intenção de continuar no país e eventualmente conseguir um Green Card para poder ficar permanentemente. Se a equipe de admissão desconfiar que você tem intenção de voltar para o seu país, você não entra.

Contudo, o maior problema do aluno estrangeiro é financeiro. Não existem bolsas para a faculdade de medicina (são pouquíssimas exceções) e os Americanos, quase em sua totalidade, pegam financiamento estudantil, o que não está disponível para estrangeiros. Esse é o motivo pelo qual muitas instituições não aceitam alunos de outros países. Em alguns casos isolados a universidade exige comprovação de condições financeiras para arcar com os cursos da faculdade de medicina (em torno de 200 mil dólares pelos 4 anos) como condição para consideração (e veja que eu disse consideração, não aprovação! Ao comprovar que pode pagar os 4 anos de curso, o candidato pode ter sua aplicação considerada, porém não há qualquer garantia de aceitação). Nesses casos, quando o aluno é aceito é necessário pagar o curso semestralmente ou anualmente. Não há mensalidade de faculdade nos EUA.

Cidadão “completo”

A escola de medicina exige do aluno uma completa formação científica e cultural prévia, por isso o Brasileiro interessado em fazer medicina nos EUA deve estrategicamente planejar um currículo escolar e de vida que possa futuramente mostrar ao comitê de admissão da med school que ele é uma pessoa “completa” e que sua formação em medicina irá torná-lo um cidadão do mundo, um líder do futuro. Sim gente, isso parece um exagero até porque a maioria dos médicos na prática nem sequer precisam da tal “liderança”, mas os comitês de admissão de medicina parecem acreditar que médicos são deuses encarnados e que os alunos precisam provar que são “semideuses” se preparando para governar o mundo do futuro!

O processo de seleção em medicina nos EUA é realmente insano e fora da realidade, mas é isso o que faz a demanda muito alta e pouca oferta. Os comitês de admissão (as pessoas que escolhem a dedo em cada faculdade quem vai entrar em seus cursos) podem se dar ao luxo de pinçarem cuidadosamente os melhores indivíduos que se candidatam. O resultado é que com o tempo convencionou-se avaliar a vida inteira da pessoa, já que em matéria de testes apenas, o que seria equivalente ao vestibular (ou ENEM) no Brasil, os alunos que se candidatam a medicina são muito parecidos. Se há apenas 100 vagas e 1000 gabaritam a prova, como você escolhe quem vai entrar? Você naturalmente começa a olhar outros fatores, como o desempenho acadêmico prévio do aluno, a personalidade dele (percebida através da entrevista pessoal), o histórico extracurricular (que pode mostrar o que esse aluno gosta de fazer, sua predisposição para ajudar os outros, seu potencial de liderança, etc.).

Para tornar sua compreensão mais fácil, vou quebrar os itens levados em conta no processo de admissão um a um, em ordem de importância:

– Desempenho acadêmico nas matérias pré medicina (premed). Seja o que for que o aluno faça antes de aplicar para entrada em medicina (é assim que se fala: apply to get into med school), ele precisa cursar 9 matérias: física, biologia, química (geral e orgânica), matemática, cálculo, inglês (não é o cursinho de inglês que você faz no Brasil, mas sim o inglês de college Americano), bioquímica, sociologia e psicologia.

Essas matérias, como mencionei anteriormente, precisam ser de nível superior (não basta o que você aprendeu no ensino médio) e precisam conter 1 ano de seus respectivos laboratórios (em biologia, química e física), ou seja, o aluno precisa cursar biologia, física e química (geral e orgânica separadamente), teórica e prática (laborátório) com 1 ano de cada (que são feitos juntos). Muitas escolas oferecem o premed como um curso especial e geralmente dura(va) 2 anos. Com as novas matérias, estima-se que o premed a partir de 2015 passe a durar 3 anos.

O aluno que planeja estrategicamente sua entrada em medicina, contudo, não precisa fazer o premed separado, ele já escolhe um curso superior que contenha essas 9 matérias ou complementa as faltantes fazendo-as junto com seus anos no college. Cursos que contêm muitas das matérias básicas são farmácia bioquímica, engenharia química, engenharia mecatrônica, engenharia elétrica, engenharia bioquímica e engenharia biomédica (muito ligada à medicina). Um colega meu fez neurociência (incomum no Brasil, mas bastante frequente como concentração em colleges nos EUA) e ele conseguiu fazer as 9 matérias. Em faculdades cujas matérias podem ser feitas de forma independente, é fácil complementar biologia e química com as matérias faltantes e estar pronto para aplicar para a medschool ao final do bacharelado. Algumas faculdades de biologia e química englobam também matemática e cálculo, psicologia e neurociência também podem englobar todas as matérias.

O aluno interessado em medicina precisa ter desempenho acadêmico PERFEITO nessas 9 matérias! As notas nessas matérias e em seus respectivos laboratórios serão criteriosamente avaliadas pelo comitê de admissão. As notas nos EUA são calculadas em uma média que se chama GPA (há calculadoras online em que você coloca suas notas no Brasil para ver qual o seu GPA), em uma gradação que vai de 1.0 a 4.0. O aluno, para ser considerado para entrada em medicina, precisa ter GPA no mínimo 3.5. Isso, na prática, significa nunca ter tirado abaixo de 8.0 em todas as matérias da faculdade. As melhores instituições como Harvard e Yale nem sequer olham para aplicações com GPA menor do que 3.7.

De qualquer forma, ao invés de fazer uma faculdade com currículo fechado no Brasil e depois ter que passar alguns anos complementando o currículo com as matérias que faltam, é melhor fazer o college (faculdade) nos EUA direto.

Lembrando que nos EUA, o aluno é quem monta sua grade curricular, então quando ele entra no college, ele pode fazer as matérias que bem entender e depois se focar numa área específica para obter o título de bacharel (chamado de major, o que seria uma “concentração” em uma área específica). O aluno também pode ter um major e um minor, ou seja, uma concentração maior e outra menor, o que enriquece seu currículo para entrada em medicina. É muito comum em alunos com vistas na medschool, se formarem em biologia como major e minor em química ou o contrário. Major em psicologia e minor em neurociência ou biologia também é comum para entrada em medicina. Mas é possível entrar em medicina tendo se formado em qualquer coisa, desde que a pessoa curse as 9 matérias premed, dentro ou fora do curso que ela escolheu no college.

Outra vantagem da pessoa fazer o college nos EUA é que 1. ela tem chance de eventualmente conseguir um Green Card, aumentando suas chances de ser aceita em medicina e 2. ter um diploma emitido nos EUA, o que também aumenta as chances de entrada – muitos comitês de medicina não “gostam” de gente que fez faculdade no exterior porque eles consideram que as faculdades nos EUA são “melhores” e eles não tem como saber se a faculdade que a pessoa fez em outro país (Brasil = país de terceiro mundo) é realmente boa.

Um pacote completo para estudar para o MCAT da Princeton Review, que é uma das melhores editoras de guias de estudo. Essa é a edição mais atual em 2021. [/caption]– Nota no MCAT (Medical College Admission Test), o “vestibular” de medicina – A ênfase para a nota do MCAT varia muito dependendo da instituição, mas é seguro afirmar que uma nota muito boa nesse teste é obrigatória para entrada, principalmente nas melhores universidades.

O MCAT é um tipo de um ENEM, um teste único, aplicado aos alunos por uma entidade ligada ao conselho federal de medicina nos EUA, portanto não é um teste aplicado pela universidade em si.

O MCAT contém as 9 matérias premed (inglês, física, biologia, química, matemática, cálculo, psicologia, sociologia e bioquímica) e mais redação e compreensão de texto. É um teste dificílimo, que visa principalmente eliminar candidatos que não tenham perfil acadêmico para suportar o rigor do curso de medicina.

Para mais informações sobre o MCAT, acesse a página oficial: https://www.aamc.org/students/applying/mcat/

– Instituição em que o aluno fez a faculdade (college) – É aí que mora o principal problema para os Brasileiros. O comitê de admissão precisa ser capaz de avaliar as notas do aluno presentes em seu histórico acadêmico frente ao que eles SABEM sobre a instituição em que o aluno fez a faculdade. Os comitês tem uma boa ideia do perfil e qualidade das instituições nos EUA e sabem que um B em neurociência na Universidade Johns Hopkins é mais valioso do que um B na mesma matéria na Universidade de Indiana. Johns Hopkins é mais difícil, por isso o aluno que consegue um B é mais inteligente e estudou mais do que o que conseguiu B na Uni. de Indiana.

Os membros dos comitês de admissão de medicina não fazem a menor ideia da gradação de qualidade nas instituições Brasileiras (ou Alemãs ou Austríacas, por exemplo). Algumas universidades Brasileiras como USP e ITA podem até ter uma certa fama, mas qualquer outra instituição, incluindo as Universidades Federais, são desconhecidas para os Americanos. Não são só os Brasileiros que sofrem com isso, é bom mencionar! Uma amigo alemão, que estudou na Universidade de Bonn, uma excelente faculdade na Alemanha não conseguiu entrar em medicina nos EUA pois os comitês não tinham parâmetros para comparar a Universidade de Bonn com as instituições Norte Americanas e sendo assim, não tinham como saber se aquele aluno era realmente bom.

O prestígio e qualidade da faculdade onde a pessoa obteve seu bacharelado é super importante para entrada em medicina e é recomendado que essa instituição seja nos EUA, como já falamos em outros pontos.

– Desempenho acadêmico – Esse item está intrinsecamente ligado ao anterior. O currículo acadêmico do aluno é avaliado individualmente. Os comitês avaliam matéria por matéria e “julgam” o quanto realmente vale a nota que o aluno obteve do ponto de vista da qualidade daquela instituição. Por isso eles precisam CONHECER BEM a faculdade em que o aluno fez o college e uma instituição em outro país perde muitos pontos. Quais matérias o aluno escolheu fazer também contam, pois isso revela os interesses pessoais do aluno (contando que nos EUA são os alunos que escolhem as matérias que querem fazer na faculdade).

– Atividades extracurriculares – É aqui que os Brasileiros “sentam” bonito! A maioria das “grad schools” como medicina, direito, odonto, arquitetura, dentre outras, querem dar suas vagas a alunos que se destacam também fora do ambiente acadêmico. A filosofia é que um médico ou um advogado que se preze é um cidadão completo, não um crânio que é muito bom na escola, mas um zero à esquerda na vida cotidiana.

Como no Brasil, entra na faculdade quem vai bem no vestibular (ou no ENEM), os alunos não costumam se envolver em muitas atividades extracurriculares, muito menos com perfil assistencial (voluntariado).

Eu trabalhei no setor de admissão nas faculdades de Harvard e posteriormente, na Inglaterra na Universidade de Oxford, no departamento que dá as famosas bolsas conhecidas como ‘Rhodes Scholarship’. Eu via com frequência as aplicações dos alunos e ficava literalmente embasbacado. Eu não podia acreditar que aquelas crianças de 20, 21 anos já tinham feito tanta coisa na vida. Se eu não soubesse que elas eram realmente jovens, eu apostaria que eram muito velhas! Muitas delas tinham feito trabalhos humanitários diretamente com a ONU, Unesco, voluntariado em hospitais, hospícios, asilos e muitas tinham experiência internacional de trabalho na África, Bangladesh, Sri Lanka e outras regiões carentes. Muitas já tinham aberto empresas próprias, inclusive ONGs, algumas tinham livros publicados e praticamente todas tinham histórico de excelência em algum esporte ou música, tendo ganho campeonatos e medalhas.

É claro que esse padrão altíssimo só é visto nas Ivy Leagues (Harvard, Columbia, Brown, Universidade da Pensilvânia, Dartmouth, Princeton, Cornell e Yale), além do MIT e Stanford e fora dos EUA, Oxford e Cambridge na Inglaterra, dentre outras instituições de altíssimo prestígio. Contudo, o aluno que deseja entrar em medicina precisa ter noção de que estará concorrendo com jovens que já sabem “qual é o esquema” desde muito cedo e sendo assim, já vem se preparando há anos para montar um currículo exemplar.

Esse currículo de atividades “não acadêmicas” deve demonstrar para o comitê de admissão que o candidato:

– tem inteligência emocional, sendo capaz de lidar com adversidade, diversos tipos de pessoas, conflitos e situações emergenciais;

– tem vontade de ajudar os outros, mesmo que não receba nada em troca;

– tem perfil de líder;

– tem um nível cultural avançado (demonstrado pela proficiência em algum instrumento musical ou esporte);

– tem potencial para enriquecer culturalmente e cientificamente a instituição que o aceitar como aluno;

– teve oportunidade de vivenciar de perto a vida e rotina de um médico (fazendo o chamado ‘shadowing’ ficando junto com um médico durante vários dias de cada vez, o expediente todo) e já voluntariou dentro de uma clínica ou hospital. Muitos alunos também fazem o chamado EMT, que é atender em ambulâncias. Posições profissionais como CNA (auxiliar de enfermagem) e scribe (documentador médico) estão se tornando praticamente obrigatórias para os alunos pré-medicina.

Gente, tudo é muito, mas muito artificial! Tendo participado de dois dos comitês de admissão mais ferrenhos do mundo, eu posso afirmar que os pais dos alunos já preparam seus filhos desde cedo para a construção desse tipo de currículo e as crianças se esforçam para fazer tudo direitinho, porém, estrategicamente, a fim de desenhar a aplicação perfeita para a faculdade que eles pretendem entrar.

Ou seja, os alunos que vão fazer trabalho voluntário na África ou que abrem ONGs, voluntariam no hospital de câncer, etc., não fazem isso porque são “almas caridosas” e indivíduos exemplares, eles fazem isso porque sabem que precisam de “itens” muito invejáveis para mostrar aos comitês de admissão como são “diferentes” dos outros. E conseguem… Por esse motivo, não importa se você gosta ou não de fazer esse tipo de coisa, seus concorrentes terão feito e se você não tiver nada para mostrar em seu currículo, isso pesará demais contra você.

Brasileiros podem se beneficiar do fato do Brasil ser um país de terceiro mundo com muitas “mazelas sociais”. Há muitas oportunidades de voluntariado e trabalho com comunidades carentes. Há muitas ONGs que aceitam voluntários jovens e emitem certificado de participação. Também é possível voluntariar em hospitais, casas que cuidam de idosos, ONGs que fazem trabalhos em favelas, com foco na saúde, educação, empreendedorismo, etc.

É importante manter em mente que tudo o que for colocado na aplicação precisa ser comprovado, então não adianta inventar que você fez isso ou fez aquilo, se não não puder comprovar não vale! Tem que inclusive fornecer telefone de uma pessoa que possa ser contatada e “conversar” sobre o seu desempenho naquela atividade.

– Cartas de recomendação – Outro item desconhecido dos Brasileiros, mas amplamente utilizado no exterior (não só nos EUA). As cartas de recomendação são cartas escritas e seladas por pessoas que conhecem bem o aluno. Podem ser ex-professores, empresários para quem o aluno tenha trabalhado, pessoas com autoridade que tenham supervisionado o trabalho do aluno em atividades voluntárias, até mesmo amigos ilustres da família que conheçam bem o aluno e aceitem escrever uma recomendação. Por exemplo, se sua família (ou você) tem contato pessoal com um político ou com um empresário conhecido, você pode pedir para esta pessoa escrever uma carta de recomendação a seu favor. Para entrada em medicina, contudo, as cartas de recomendação (LORs) devem obrigatoriamente ter sido escritas por ex-professores de matérias RELEVANTES (ou seja, professor de filosofia ou de português não conta), médicos com quem o aluno tenha feito “shadowing”, supervisores de estágios e voluntariado em clínicas e hospitais ou pelo diretor do comitê de ajuda aos alunos que querem entrar em medicina da instituição onde o aluno fez o college (departamento obviamente que só existe nos EUA).

As cartas de recomendação costumam “desempatar” alunos que tem notas e perfil extracurricular similares.

– Essay – O Essay é um artigo (ou uma “redação”) que você mesmo deve escrever com o objetivo de se apresentar ao comitê de admissão, explicando quem você é, um pouco da história da sua vida e por que você deseja fazer medicina.

O essay é levado muito a sério e para alunos cuja língua mãe não é inglês, é recomendado pagar um revisor profissional para corrigir os erros e dar recomendações para que o texto em inglês fique perfeito. Um único erro ou frase mal escrita em inglês pode fazer com que toda a aplicação do aluno seja descartada. As universidades de medicina têm tolerância zero para com erros de inglês, o que passa a mensagem de que o aluno é relaxado, descuidado e indisciplinado – características indesejáveis em um médico! Você terá muito tempo para escrever o essay, já que é algo que você escreve por conta e envia junto com toda a documentação da aplicação, pelo correio ou online, como muitas instituições tem feito nos últimos anos. Muitos alunos Americanos contratam consultores de entrada em medicina e têm todo o seu material, incluindo essays, minuciosamente revisados e corrigidos múltiplas vezes pela equipe do consultor. Esses consultores, inclusive, prestam aconselhamento quanto ao currículo do aluno. Se eles acreditam que o aluno não tem experiência profissional ou voluntária suficiente na área médica, eles indicam o que fazer para melhorar esse aspecto da aplicação.

Um caso que vi esses dias (aqui nesse link) conta a história de um rapaz formado com louvor na Universidade de Nova York em química orgânica (“o” assunto mais difícil do currículo premed), com vasta experiência como professor de biologia e química. Ele complementou suas matérias premed, tudo direitinho, tirou uma nova muito boa no MCAT, construiu um currículo com atividades extra-curriculares e finalmente aplicou para a med school. Não foi aceito. Ele então partiu para o Nepal e para a Uganda, onde ele fez trabalhos voluntários diretamente com equipes de ajuda humanitária. Você acha que com isso então ele deveria ter sido aceito, não? Pois é, mas não! Depois dessa segunda recusa, ele contratou um consultor que lhe recomendou pegar ainda mais experiência lidando diretamente com pacientes. Agora ele dirige uma ambulância e está se preparando para o terceiro round de aplicações.

– Nota do TOEFL (ou teste similar) – Nos EUA costuma-se exigir o TOEFL para todos os alunos cujo ensino médio ou faculdade não tenha sido cursada em país de língua inglesa, mas outros testes similares são aceitos também. A nota do TOEFL para entrada em medicina precisa estar perto do topo, já que há muitos alunos estrangeiros que tentam entrar em medicina e o nível de inglês da maioria é fluente.

– Entrevista pessoal – Depois de passar por todo esse processo de seleção inicial, o comitê afunila os alunos ‘desejáveis’ em uma lista menor e chama esses alunos para uma entrevista pessoal e individual, bem aos moldes de uma entrevista de emprego. Evidentemente, o aluno estrangeiro precisa se comunicar muito bem em inglês, demonstrar autoconfiança, caráter, firmeza e estar pronto para responder às perguntas do entrevistador, que giram em torno do histórico de vida do candidato e os motivos que o levam a desejar ser médico. Evitar clichés como “eu sempre quis ajudar os outros” ou “eu sempre sonhei em salvar vidas” é ultra recomendado! O entrevistador não é besta! A realidade é que as pessoas não fazem medicina porque são almas caridosas que querem salvar os outros! Se você quer fazer medicina, mas não consegue justificar sua motivação além de clichês batidos, faça uma autorreflexão bem séria e pense se é isso mesmo o que você quer da vida e porquê. Serão esses argumentos muito bem elaborados, pensados e (bem) longe do óbvio que poderão convencer a equipe de admissão que você deve estudar medicina com eles.

Se você me fizer perguntas abaixo, procure me dar detalhes como a sua idade (ou do seu filho/a), desempenho escolar e porque deseja fazer medicina nos EUA (para voltar ao Brasil depois ou para morar lá permanentemente).

Compartilhe!
Cadastre-se gratuitamente no Guia da Vida e tenha acesso a nosso melhor material sobre desenvolvimento pessoal, produtividade, saúde mental e motivação:


156 comentários em “Como estudar medicina nos Estados Unidos?”

  1. Minha irmã está fazendo medicina na Universidade da California (UCLA) e eu vou contar pra vocês… ela se preparou durante 6 anos para entrar! É muito, mas muito difícil mesmo. Este artigo é o mais fidedigno e completo que eu já li e realmente diz como é e enfatiza a dificuldade para Brasileiros (ou qualquer estrangeiro) entrar em medicina nos EUA.

    A grande maioria (praticamente todas) as faculdades de medicina nos Estados Unidos NÃO ACEITA ESTRANGEIROS! Então se você não tem um plano muito bem bolado de como você vai conseguir um Green Card bem antes de aplicar para a entrada na univerdade, esqueça, você não vai conseguir. E nas que aceitam estrangeiros, a pessoa precisa pagar a faculdade TODA à vista! Contudo, mesmo nas poucas que aceitam estrangeiros, essas vagas ficam com pessoas de outros países de língua inglesa como Inglaterra, Irlanda, Austrália e Nova Zelândia. Estrangeiros não conseguem bolsas nem financiamento, então, a não ser que você seja mulionário, você não vai poder pagar a faculdade de medicina!

    A pessoa que quer fazer medicina nos States precisa traçar um planejamento sério e bem pesquisado pois a construção da aplicação de um aluno demora anos (mesmo os alunos Americanos). Não é que nem no Brasil que é só o cara fazer vestibular, passou, entrou. Como explicou o artigo, o aluno precisa trabalhar muito nos anos anteriores à entrada em medicina para tecer um currículo de atividades voluntárias, estágios em clínicas, atendimento direto em hospitais, emergências e ainda ter elementos que os destaquem dos outros milhares de estudantes que tem currículos parecidos.

    Também não adianta fazer faculdade no Brasil e querer aplicar direto, a pessoa precisa ter créditos feitos em instituição Norte Americana. Além disso, o MCAT que é o “vestibular de medicina” é extremamente difícil e cobra conteúdo de nível superior, ou seja, o que o povo acha dificil em química, física e biologia para fazer o vestibular no Brasil é piada no MCAT. Você precisa saber tudo isso (o conteúdo do vestibular) mais o que um aluno de faculdade de física, química e biologia aprenderia. Sem contar as outras matérias (sociologia, bioquímica, psicologia, matemática – o que inclui cálculo) e é óbvio, o inglês mais do que perfeito (o inglês do aplicante para medicina precisa ser “melhor” do que o do Americano comum.

    É importante artigos como este para dar um banho de realidade nos sonhadores de plantão que ficam procurando no Google sobre como fazer medicina nos EUA… A maioria das pessoas jamais vai conseguir, por diversos motivos. Não é uma questão de aguar o sonho dos outros não, é simplesmente um nível de complexidade e dificuldade que é totalmente estranha para o aluno Brasileiro, sem contar na necessidade de conseguir o visto de permanência (green card) para ser capaz de aplicar para medicina…

    Eu segui o caminho da minha irmã que foi diligentemente focada nesse objetivo por 6 anos, tendo colocado todos os pontos no papel, plenejando estrategicamente como ela iria conseguir chegar lá.

    Posso deixar como dica para o pessoal que REALMENTE está interessado em saber os detalhes do processo de entrada em medicina nos estados unidos que consulte o forum: forum.studentdoctor.net É uma comunidade de estudantes de medicina e alunos pré medicina. É possível descobrir muitos “truques”, funcionamento do processo e principalmente ter uma ideia de como é difícil e árduo o caminho para chegar lá mesmo para os próprios americanos.

    Se você realmente está disposto a se focar só nisso e nada mais pelos próximos 4 a 6 anos (até entrar na faculdade de medicina), depois 4 anos de faculdade e + 4 a 9 anos de residência, sabendo que provavelmente terá que se casar com um americano para conseguir um green card e permanecer nos EUA por todo esse tempo sem voltar para o Brasil (a faculdade de medicina NÃO TEM FÉRIAS!), residência não tem férias, é ficar lá até ser médico formado com residência. Topa a parada? Vá em frente!

    Responder
      • Nossa, nunca mais visitei este site, não tinha visto sua pergunta! Minha irmã se casou com um Americano. não por interesse, é claro, eles estão casados há 10 anos e são muito felizes. Ela planejou tudo nos mínimos detalhes, ela costumava dizer,”bom, eu vou casar mesmo, não vou? Solteirona é que eu não vou ficar! Então se vou casar, vou casar com um Americano e viver o meu sonho!” E foi exatamente isso o que aconteceu!

        Responder
  2. Bom, já sou estudante de medicina no Brasil.
    Gostaria de saber se vcs tem alguma informação sobre o processo de residência nos EUA.
    Já andei pesquisando e vi que são vários “steps”, que é bem caro todo o processo.

    Alguma informação realista/pé-no-chão, como os posts sobre os processos de “aplicação” para o curso de graduação?
    Obrigado.

    Responder
    • Victor,

      Sim, os steps são as provas que devem ser feitas como parte do processo de revalidação (é o mesmo que os próprios alunos nas faculdades Americanas fazem). São 3 provas, 2 teóricas e a última prática. O ideal é prestar os steps durante a faculdade para não acabar ficando muito difícil relembrar o conteúdo depois de anos. Tem gente no Brasil que dá aulas de cursinho pros steps e tem informação de como prestar as provas, se você procurar no Google, acredito que possa levantar bastante informação sobre como fazer estes testes.

      Quanto à residência, é mais difícil conseguir uma vaga tendo feito faculdade no exterior, mas é possível. Quanto mais altas suas notas nos 2 primeiros steps (o terceiro só é feito após o final do primeiro ano de residência) e uma média de notas na faculdade (GPA) sólida (acima de 3.8), você se torna competitivo para entrar em uma residência.

      Esse é processo necessário para “revalidação” do diploma Brasileiro nos EUA. É a única forma de praticar medicina no país tendo feito faculdade no exterior, não sei é essa a sua intenção…

      Quanto ao valor, não sei se é muito caro, não… O valor para inscrição nos steps não é caro e você não é obrigado a fazer cursinho, só se quiser.

      Abraços,

      Diego

      Responder
        • Olá Nivaldo,

          Acho que sua pergunta é um pouco antiga, mas caso ainda esteja interessado no processo de entrada em residência nos EUA, vou te responder.

          O processo é unificado. Você faz a aplicação para a residência no conselho federal de medicina dos EUA, não para a escola X ou Y. Você marca na aplicação quais as escolas de sua preferência em ordem. Todas as escolas juntas pegam essas aplicações e vão chamando os alunos em que estão interessadas para entrevista pessoalmente (bem estilo entrevista de emprego). No final do processo ocorre o “match” que é o emparelhamento dos alunos com as escolas que os escolheram. Os alunos que tiveram maior nota nos STEPs do USMLE tem mais poder de escolha. Alunos internacionais têm menor poder de escolha. Os alunos que não forem escolhidos são remanejados em outro processo chamado “scramble” onde as vagas restantes em todas as escolas juntas são preenchidas com esses alunos (que não têm escolha sobre onde ir). Ou seja, sua primeira escolha pode ser Harvard, mas você pode acabar no Alaska!

          Abraços,

          Elisângela

          Responder
    • Altamente recomendado prestar os STEPs 1 e 2 durante a faculdade. Se você deixar para o final, vai acabar esquecendo conteúdo e ter uma nota menor. A nota dos STEPs 1 e 2 para que estrangeiros entrem em uma residência precisa ser muito altas! Precisa ser uma média acima dos alunos Americanos e de faculdades fora dos EUA que costumam mandar alunos para as residências (como as Caribenhas). O que eu fiz foi comprar livros (na Amazon em formato Kindle) e baixar torrents de livros texto, além de livros escritos para estudar para os STEPs e comecei a estudar desde o primeiro ano de medicina no Brasil. O conteúdo exigido nos EUA é muito mais detalhado e profundo do que o que vemos no Brasil, além da barreira da língua e ter que memorizar termos em Inglês que soam completamente diferentes!

      Eu não tinha vida social, apenas um namorado muito compreensivo! Como temos férias no Brasil (a faculdade de medicina nos EUA não tem férias!), viajava para os EUA nesses intervalos para fazer cursos e melhorar meu Inglês (aproveitei bastante a Harvard Summer School e a Harvard Extension). Eu tinha um ritmo bem intenso de estudo e estudar simultaneamente em inglês e português teve um impacto super interessante. Muitas vezes, em provas eu lembrava de termos, definições e conteúdo porque eu tinha estudado o mesmo em inglês e aquilo tinha reforçado a memória.

      No final das contas, prestei o STEP 1 e tirei um score altíssimo (268), muito mais alto do que a média. No STEP 2 eu tirei 270, também muito alto. Eu fui chamada para diversas entrevistas de residência exclusivamente por causa desses scores! Fui aceita em 5 programas de residência, todos top e escolhi a Duke University.

      Você deve prestar o STEP 1 no final do segundo ano e o STEP 2 no final do quarto ano. O STEP 3 é feito no final do primeiro ano de residência.

      Eles não são caros, cuidado com o que você pesquisa sobre esse assunto no Brasil! Tem muita gente vendendo cursos para os STEPs, e prometem fazer todo o processo, inscrição, etc. Você não precisa disso! É um mero testinho. É só você mesmo pesquisar, se inscrever (a inscrição é barata) e pode fazer os testes no Brasil mesmo.

      Se você já deixou passar o STEP 1, não tem problema.Você não é obrigado a prestá-lo no final do segundo ano.

      Abraços e boa sorte!

      Elisângela

      Responder
        • Oi Kátia,

          Puxa, não visitei mais este site, não tinha visto que tinha uma pergunta aqui pra mim! Faço residência em Neurologia na Duke University (fiz medicina no Brasil).

          Abraços,

          Elisângela

          Responder
        • Oi Nicolli,

          O processo de revalidação do diploma nos EUA é a própria residência. Se a pessoa quer praticar medicina nos EUA ela é obrigada a fazer a residência no país, mesmo que já tenha eventualmente feito residência no Brasil ou em outro país. Além da residência, a revalidação do diploma exige 3 testes chamados STEPs do USMLE (o Diego já falou bastante sobre esses testes nas respostas aqui e no artigo sobre medicina em Harvard). As duas primeiras provas devem ser prestadas DURANTE o curso de medicina e a terceira após o primeiro ano de residência (que é o internato no Brasil). Essa terceira prova é prática e exige um inglês perfeito. Essas provas são MUITO difíceis! Elas são muito mais difíceis que o revalida no Brasil e por isso a pessoa deve prestá-las durante o curso de medicina e não deixar para depois, pois se esquece muita coisa depois do curso. Tendo feito residência nos EUA, que não é um curso, mas sim um emprego propriamente dito, a pessoa já tem visto de trabalho e ao terminar a residência é natural que consiga um emprego facilmente já no hospital onde está fazendo a residência. Existe também o Fellowship que é mais uma especialização após a residência que muitos estudantes decidem fazer. De qualquer forma, não há qualquer dificuldade de conseguir emprego se você resolver ficar e trabalhar nos EUA após a residência. Há uma carência muito grande de profissionais de saúde no país e muitas vagas abertas.

          Boa sorte para você, desejo que seu sonho dê certo!

          Elisângela

          Responder
      • Elisângela, quais foram as outras universidades que vc passou ? Pq quero muito fazer a residência em Harvard ou Yale, vc acha que é possível?

        Responder
        • Olá Samuel,

          Além da Duke, que é onde faço a minha residência, passei na University Of Pennsylvania, University Of Wisconsin, University of Washington (Seattle), e na University of North Carolina (Chapel Hill). As chances de entrar em Harvard ou Yale são mínimas. Pense que quem chega ao final da faculdade de medicina já é extremamente capaz e inteligente, e todos querem essas escolas. Na peneira de seleção, acabam indo para Harvard, Yale, Stanford, MIT, Johns Hopkins somente as “estrelas”. O diferencial não é somente em notas, pois notas altas todos têm. Uma colega que foi chamada por Yale fundou uma ONG aos 17 anos, passou 3 anos na África trabalhando em uma ONG contra a infibulação feminina, foi líder de classe em todos os anos da faculdade, trabalhou na fundação Clinton e tinha uma carta de recomendação pessoal do presidente Bill Clinton. Esse é o “perfil” do aluno que é chamado por essas escolas e acredite, eles não são tão raros! Isso significa que essas escolas absorvem todas essas “estrelas” e o “resto” é distribuído entre os “outros” hospitais-escola. Lembrando que o processo de seleção da residência é generalizado, até certo ponto você não pode escolher para onde vai.

          Abraços,

          Elisângela

          Responder
          • Entendi, será que vc poderia me dar dicas de como se preparar para os steps ? (Já falo inglês quase fluente, digo, avançado)

          • Olá Samuel,

            Eu não sou médico, não prestei os STEPs! O conhecimento que eu tenho é por ter trabalhado no departamento de seleção de alunos. Mas o que posso te dizer é que há muita, muita coisa na internet sobre esse assunto. Há centenas de livros na Amazon, que você pode comprar em formato digital (Kindle). Há sites criados só com o objetivo de ajudar os alunos na preparação para os STEPs. Há treinamento de diversos tipos (comunidades, vídeos, etc.). É só você procurar, mas em inglês, é claro!

            Abraços,

            Diego

    • Demora uns 3 a 4 anos, incluindo tradução de diploma, testes e mais testes e prova oral, fora uma “reabilitação” porque muitas doenças que você tem no Brasil não se tem lá.

      Responder
  3. Sabe as diferenças práticas principais do ensino de medicina no Brasil em contraste com o dos EUA? No tocante ao aprendizado, a preparação, infraestrutura…

    Responder
    • Olá Gabriel,

      Sua pergunta é antiga, mas já que estou passando por aqui, vou responder! O ensino da medicina no Brasil é muito parecido com os EUA, contudo, como os Americanos são mais detalhidas e mais organizados, o conteúdo é cobrado de forma mais intensa e profunda. A diferença mais significativa é o tempo de estudo. Enquanto no Brasil os alunos permanecem na escola de medicina por 6 anos, nos EUA o curso dura apenas 4. Por que isso? Porque nos EUA, para o aluno entrar em medicina, ele precisa já ter um curso superior (medicina é “pós graduação”) e sendo assim, ele já tem uma boa bagagem científica (através das matérias pré médicas exigidas para entrada no curso) para acompanhar o curso e isso dispensa 2 anos de estudo.

      Abraços,

      Elisângela

      Responder
  4. Olá
    Eu quero fazer medicina nos EUA e sei que primeiro tenho que fazer o college, para depois fazer a pós que é a graduate lá. Mas caso eu queira me especializar em pediatria quando terminar a faculdade o que faço? A especialização pode ser feita lá também? Ou na graduate a gente ja escolhe a área que deseja se especializar?

    Responder
    • Olá Ester,

      Em primeiro lugar, você tem que saber que precisa preferencialmente fazer o college nos EUA e nesse período dar um jeito de consguir um Green Card e posteriormente cidadania Americana, pois as faculdades de medicina geralmente não aceitam alunos estrangeiros.

      Pediatria é uma residência, não é uma especialização. Você entra na residência pós a faculdade de medicina. Fazer residência nos EUA é pré-requisito para exercer medicina no país, então fazer nos EUA ou no Brasil depende dos seus planos para o futuro. Você quer fazer faculdade nos EUA e voltar para o Brasil para praticar ou você quer fazer faculdade lá porque deseja imigrar e viver nos EUA?

      Abraços,

      Diego

      Responder
  5. Primeiro: Obrigada por responder rsrs.
    E sim, eu quero voltar para o Brasil, pelo menos até agira é o que pretendo. Nesse caso precisarei primeiro revalidar meu diploma para fazer a residendia em pediatria aqui no Brasil?

    Responder
    • Olá Ester,

      Se você quer voltar para o Brasil, não vale a pena. É muito mais difícil entrar em medicina nos EUA e a revalidação do diploma (sim, você precisaria fazer) é dificílima.

      Abraços,

      Diego

      Responder
    • Se vale a pena “esse esforço” ou não, isso depende muito dos seus objetivos de vida. Eu sempre quis morar e trabalhar nos EUA, mas vi de perto parentes que foram antes de mim e que, sem uma formação acadêmica sólida, acabaram tendo que fazer trabalhos como limpeza doméstica e trabalhar em restaurantes. Vendo isso desde pequena, eu comecei a pesquisar quais profissões eram sólidas o suficiente para que me dessem a segurança, a satisfação e o retorno financeiro que eu buscava. Eu tive parentes advogados, psicólogos, administradores que se deram mal nos EUA. Eu cheguei à conclusão de que o rol de profissões realmente seguras é muito pequeno e se resumem à formações técnicas como engenharia, odontologia, medicina e arquitetura. Como eu não tenho nenhuma vocação para mexer com essas áreas exatas (engenharia, arquitetura), escolhi medicina. Pesquisei muito, planejei nos mínimos detalhes como exatamente eu iria dar conta de fazer o que eu queria e fiz. Hoje estou no quarto ano de medicina na Universidade de Chicago, considerara uma das melhores nos EUA.

      Minha dica pra você é pensar no que você realmente deseja alcançar na vida e se desligar um pouco da questão do “fazer o que você gosta”, que é uma grande armadilha. Você deve fazer algo que te dê satisfação sim, mas em primeiro lugar, é importante escolher uma profissão que lhe dará a vida que você quer viver. No final das contas, uma profissão rentável e segura vai lhe dar muito mais satisfação do que algo que você curtiu fazer na faculdade, mas que não põe comida na mesa!

      Abraços e boa sorte!

      Marlene

      Responder
      • Olá Marlene, gostei muito do seu comentário e tenho o mesmo ponto de vista que você, gostaria de conversar com você, pois tenho muitas dúvidas, será que você poderia me passar o seu e-mail ou eu passar o meu?

        Responder
      • Olá Marlene, tudo bem? Minha filha terminou o ensino médio no Brasil. Ela quer fazer medicina e tem cidadania americana (nasceu ai). Você poderia enviar algumas informações de como ela pode entrar num college aí nos EUA? Muito obrigado.

        Responder
        • Olá Jonecir,

          Há muitos sites que dão essas dicas. Um que recomendo é o College Confidential (só digite no Google que você acha), especial para medicina o maior deles é o Student Doctor Network. Há também muitos ebooks na Amazon que contêm todos os “segredos” da aplicação para uma faculdade.

          É importante primeiro de tudo determinar quanto de dinheiro você está disponível para desembolsar com a faculdade – algumas podem ser caríssimas. Se você não tem condições financeiras, o ideal é uma State School que geralmente tem anuidades mais baratas que podem ser interamente pagas com financiamento estudantil e bolsas. Se ela é uma aluna A, ela pode indicar na aplicação que precisa de 100% de bolsa se for aceita. Se ela não é A, ela pode conseguir “bolsinhas” pequenas (externas) que juntas ajudam a pagar pelo curso. Essas bolsas se chamam scholarships e grants e também há muitos sites e livros dedicados à ensinar como consegui-las.

          Ela também pode fazer o college no Brasil (é equivalente à qualquer faculdade). O que
          é importante para entrar em medicina é que além de notas perfeitas (sempre acima de 9) durante toda a faculdade, ela precisa acumular experiência prática na área da saúde e também em pesquisa científica. Isso ela consegue voluntariando nos laboratórios da faculdade, sendo no Brasil ou nos EUA. Experiência prática ela consegue voluntariando em clínicas e hospitais. É importante que vocês façam um plano, pois a aplicação para medicina é um quebra-cabeças enorme. São muitas “partes” que ela precisa preparar ao longo de muitos anos para que termine o college esteja pronta para prestar o MCAT (vestibular de medicina) e seja competitiva o suficiente para conseguir entrevistas e possivelmente ser selecionada.

          Boa sorte!!

          Marlene

          Responder
  6. Ola, primeiramente quero elogiar site com tantas informações úteis, específicas e leais.
    Não sei se minha pergunta já foi feita, mas a dúvida é a seguinte:

    Tenho curso superior na área de humanas e agora conclui meu primeiro ano de psicologia numa segunda graduação. Vou embora para EUA por conta do emprego do meu esposo então quero saber se entendi direito …. Como tenho 1 curso superior em outra área eu poderia aplicar direto pra psicologia (graduate) sem ter q fazer a under?

    Obrigada antecipadamente pela resposta.

    Responder
    • Olá Thali,

      Me desculpa, não tinha visto sua pergunta por aqui. Sua faculdade no Brasil JÁ É undergraduate! Se você quiser formar em psicologia nos EUA você precisa fazer doutorado na área (graduate school) e neste caso, seu diploma já conta. Contudo, como doutorado não é um mero “curso”, mas sim um emprego em pesquisa, você pode ter dificuldade para ser aceita em um curso relativamente bom só com um diploma de undergraduate. O mais importante para ser aceita em uma escola graduate é ter já publicado pesquisa científica durante a sua faculdade no Brasil, o que não é comum. Isso acaba sendo uma grande desvantagem, pois os alunos Americanos que já tem olhos na graduate school já começam a se envolver com pesquisa científica desde o primeiro ano da faculdade. Se esse não foi o seu caso, talvez fosse indicado fazer algum curso de pós graduação já nos EUA que teria dois objetivos: provar sua capacidade de seguir um currículo em inglês e com o rigor das escolas americanas, e te dar oportunidade de participar de pesquisas científicas e conseguir publicar pelo menos como quarto, quinto, sexto autor em um journal importante. Há vários sites que se dedicam a discutir através de foruns a entrada na graduate school. Te indico o Grad Cafe que tem bastante gente nessa área de psicologia.

      Abraços,

      Diego

      Responder
  7. Olá, tenho 18 anos e já faço farmácia bioquimica aqui no Brasil.
    Eu posso tentar medicina com essa formação ou terei que fazer outra graduação nos Estados unidos? Caso a resposta seja sim, ele avaliam somente as notas da minha graduação ou as do ensino médio contam também?

    Responder
    • Olá Sinara,

      Essa graduação (que nos EUA se chama undergraduate ou “sub” graduação) é suficiente, contudo, não é fácil entrar em medicina nos EUA tendo feito faculdade em outro país. Talvez fosse mais indicado fazer medicina no Brasil e depois fazer residência nos EUA para poder praticar lá.

      Abraços,

      Diego

      Responder
  8. Olá, Diego!

    Primeiro quero parabenizá-lo e agradecê-lo pela qualidade de suas informações, elas me foram muito úteis, pois tenho uma filha de 16 anos que além de ser uma aluna exemplar, é vice-campeã brasileira de esgrima já por dois anos, e esse ano de 2015 ganhou prata no Torneio Brasileiro de Equipes categoria adulto, tudo indica que terá uma carreira promissora nesse esporte. Ela gostaria muito de fazer medicina com especialização em neurocirurgia nos EUA, contratamos uma empresa de assessoria aqui de São Paulo que procurará uma bolsa esportiva e acadêmica para o College, uma vez que ela se destaca como aluna também. Porém, estou vendo que sem o Green Card ela não terá muitas chances de cursar o Graduation em medicina ou obter a sua residência, além de arcarmos com o pagamento de todas as anuidades à vista. Acredito que ela se qualificaria, por ser tremendamente determinada e ter todos os perfis necessários, porém sempre haverá o risco de não obter o Green Card.
    Por você ter também trabalhado na admissão em Oxford, lhe pergunto: temos a cidadania italiana, uma outra opção seria ela ir pra a Inglaterra estudar medicina, sei que para ter o desconto de cidadão europeu na universidade inglesa você tem que ter morado pelo menos 3 anos na Inglaterra. Estamos pensando na University of Manchester, poderia ela entrar no Pre-medical Entry de 6 anos, sem a bolsa do cidadão UE e depois de três anos morando lá, aplicar o desconto de cidadão europeu? Voce acha qual opção mais interessante: EUA ou Inglaterra? Ela não pretende voltar ao Brasil. Outra pergunta, se ela fizer medicina na Inglaterra, seu curso é reconhecido nos EUA?

    Muito obrigada por sua atenção!

    Responder
    • Olá Ana Luisa,

      Acredito que o mais importante seja ela definir qual a meta de longo prazo que ela tem. Onde ela quer morar? Nos EUA ou na Inglaterra? Ou tanto faz? Um diploma de medicina de uma univerdade na Inglaterra vale tanto quanto um de uma universidade no Brasil nos EUA, ou seja, os critérios são os mesmos. Ela precisaria prestar os steps do USMLE durante a faculdade e para praticar medicina nos EUA, não importa onde ela faça faculdade, ela precisa fazer residência no país. Nesse caso, ela poderia fazer medicina no Brasil mesmo ou em outro país da Europa, não precisa ser a Inglaterra. Ela não teria qualquer benefício ao fazer o curso no Reino Unido se o que ela quer é no final das contas, imigrar para os EUA. O custo de vida na Inglaterra é altíssimo, por isso não sugeriria essa estratégia caso não fosse a vontade dela realmente se estabelecer permanentemente neste país. Se ela tem como conseguir bolsa em college nos EUA por excelência no esporte, eu indicaria essa rota, já que ela ainda está muito longe de poder entrar em medicina nos EUA se ela seguisse por esse caminho. Até lá muita água pode rolar debaixo dessa ponte! Ela estando nos EUA cursando o college, que levaria no mínimo 4 anos, mais o pré-med, que a partir de 2015 leva 3 anos, ela pode eventualmente conseguir um Green Card via trabalho ou casamento.

      Abraços,

      Diego

      Responder
  9. Olá!

    Gostaria de fazer duas perguntas. A primeira é qual o valor aproximado para a MedSchool e a segunda é se alunos de Odontologia aqui no Brasil tem alguma vantagem na pré-admissão.

    Obrigada!

    Responder
    • Olá Emanuélle,

      Os valores variam bastante, mas a média é 40.000 dólares por ano, tirando custos de vida, só a faculdade. O valor é sempre cobrado “por ano” à vista.

      Eu não saberia dizer se você tem benefício por ter feito odonto, provavelmente não. Isso porque odontologia aqui nos EUA é curso de pós-graduação (chamado apenas de “graduacão”), assim como medicina. A forma como as instituições olham cursos de “sub-graduação” em uma área que nos EUA é “graduação” (pós), eles acreditam que o curso “só pode ser baixa qualidade”. Isso porque (isso no raciocínio deles), se nós aqui exigimos que o estudante tenha toda uma base científica, cultural e acadêmica (obtida nos 4 anos de college) antes de cursar odontologia, um curso de um país de terceiro mundo em que uma “criança” de 17 anos recém saída do ensino médio pode entrar e cursar, sem qualquer embasamento, só pode ser inferior aos nossos cursos. Novamente, isso no pensamento deles!

      Eu sofri esse tipo de preconceito aqui, tendo feito administração, que é um curso obviamente muito menos complexo do que odonto. Mesmo assim, ao me candidatar à “graduate school”, eles olharam com bastante desconfiança para o meu diploma de bacharel em administração, como se fosse um curso muito inferior porque eu pude entrar direto do ensino médio, sem ter feito college antes, como os Americanos fazem.

      De qualquer forma, você fez um curso que pode ter contido já matérias pré-médicas, isso jáé uma grande coisa que lhe economizará tempo se você realmente deseja seguir em frente e fazer medicina nos EUA.

      Abraços,

      Diego

      Responder
  10. Olá! Tenho dupla cidadania brasileira e americana. Me formei em direito e fiz o curso de piano simultaneamente.
    Hoje moro no Brasil. Sou advogada e pianista. Como pianista venci vários concursos internacionais. Mas gostaria de retornar e estudar medicina nos Estados Unidos.
    Como tenho cidadania americana e conclui curso de especialização em direito, tem algum valor? Muito obrigada.

    Responder
    • Olá Karine,

      É possível, mas você teria um longo caminho pela frente, pois tendo feito direito, você não tem nenhum dos pré-requisitos (talvez sociologia?) para entrar em medicina. Esses pré-requisitos são o que chamam de “pre-med” e são (a partir de 2015) 8 matérias: física, química, biologia, matemática, cálculo, bioquímica, psicologia e sociologia. Matemática e cálculo não são extritamente exigidas, mas são pré-requisito para física e química e são cobradas no MCAT (o vestibular de medicina). Todas essas matérias precisam ser de nível superior, então não adianta ter o conhecido adquirido no ensino médio.

      Se você está disposta a se mudar para os EUA para fazer o pre-med, uma boa opção é a Harvard Extension School, cujo programa pré-médico é aberto, ou seja, você não precisa passar por processo seletivo (em outras univerdades precisaria e tendo feito direito, uma área completamente diferente, você provavelmente não seria aceita). Durante os anos do pre-med (2 ou 3 dependendo de quantas matérias você fizer por semestre), você deve montar um currículo de atividades extra-curriculares na área de saúde. Muitos alunos voluntariam em hospitais, trabalham em ambulâncias (EMT), fazem estágio em clínicas médicas, etc. Sem esse currículo você não tem chances de entrar (mesmo). Uma coisa para começar a pensar desde agora é como você responderá na entrevista, se chegar até lá, porque resolveu mudar de direito para medicina. A faculdade de medicina nos EUA não é muito simpática com quem muda de carreira e a pessoa precisa fornecer uma justificativa muito convincente (dizer que sempre quis ajudar os outros e salvar vidas não é uma, só para mencionar!).

      Sendo Americana, você tira o maior obstáculo que os leitores desse site encontram: não ter como conseguir financiamento. Você tem, então vá em frente, mas crie uma estratégia muito bem montada, pois o caminho é longo!

      Abraços,

      Diego

      Responder
    • Só pra comentar… não sei há quanto tempo você fez essa pergunta, mas o fato de você ser pianista não significa muita coisa para entrar em medicina não… Essa coisa de “aculturamento” é importante para os Americanos na entrada do college, a primeira faculdade. Cursos profissionalizantes como medicina e direito não se importam com isso, mas sim com o nível de envolvimento que você tem na área. Como o blogger respondeu, é importante fazer muito voluntariado e até mesmo trabalhar profissionalmente na área da saúde como atendimento de emergência, auxiliar de enfermagem, Médico sem Fonteiras e outras organizações não governamentais de ajuda humanitária. Enfim, o que eles querem ver é que a pessoa pos a mão na massa para ajudar os outros. Piano é um instrumento muito solitário e egocêntrico. Do ponto de vista do comitê em medicina, eles veriam com bons olhos um esporte ou arte praticado em grupo, pois aí a pessoa estaria provando capacidade de trabalhar em equipe (importante em medicina), enquanto o piano não mostra nada muito significativo, além de persistência e foco, o que também são características importantes, mas considerando o atual mercado competitivo nas faculdades de medicina, significa muito pouco.

      Responder
  11. Boa tarde, eu sou advogada aqui no Brasil mas desisti da profissão e resolvi seguir um sonho antigo de ser médica. Como já morei nos EUA (intercâmbio) e quero morar lá, tenho pesquisado sobre o assunto. Eu teria alguma vantagem por isso,? Tenho pensado em fazer contato com alguma universidade pra saber maiores informações, seria um caminho? Você me indicaria alguma?
    Obrigada desde já!

    Responder
    • Olá Natália,

      Você diz que já morou nos EUA, mas você tem Green Card ou cidadania? Sem isso, não tem como estudar medicina, pois a grande maioria das instituições não aceitam alunos estrangeiros. As poucas que aceitam exigem que o aluno pague o curso todo à vista, mas mesmo assim não seria fácil entrar. Há muito preconceito por parte dos comitês de admissão em medicina com relação a alunos estrangeiros. Até mesmo os que possuem Green Card ou são naturalizados enfrentam dificuldades devido ao sotaque ou “aparência não Americana”. Isso ocorre pois há pressão dos pacientes que “preferem” médicos “cara de Americano” e que falem Inglês sem sotaque. O fato de que você está mudando de carreira para outra completamente oposta dificulta mais ainda o processo, pois em uma entrevista – se você chegasse a ser chamada – o comitê iria querer uma resposta muito lógica que explicasse por que só agora você quer ser médica. Dizer que é “o seu sonho” não basta! Vale mais a pena fazer medicina no Brasil e depois vir fazer residência aqui para validar seu diploma. A entrada na faculdade em si possui inúmeras barreiras, muitas preconceituosas e sem sentido, mas é assim que funciona. Indico que você consulte o site Student Doctor .net e seu respectivo fórum para entender melhor como esse processo todo funciona.

      Abraços,

      Diego

      Responder
    • O fato de você ter morado nos EUA não significa nada. Entrar na med school já é extremamente difícil para quem mora no país e tem residência. Também entrar em contato com universidades é inútil, eles só vão te dizer que você tem que se coçar para fazer os pré-requisitos (matérias premed), construir um currículo de atividades de voluntariado e trabalho remunerado na área médica, e então depois de alguns anos, se candidatar a uma vaga.

      Sinceramente, eu acho que as suas chances de entrar em medicina nos EUA são praticamente zero. Como o Diego disse, se você não tem Green Card, nem é naturalizada Americana, não adianta nem sonhar. Eu sou naturalizada e estou morando em Boston há 2 anos construindo minha aplicação para me candidatar a uma vaga e ainda tenho um longo caminho pela frente. Se você realmente quer muito isso, se informe primeiro pois sua pergunta indica que você não sabe absolutamente nada sobre como tudo isso funciona. O site que o Diego indicou é altamente recomendado para todos os que querem entrar em medicina nos EUA.

      Não me leve a mal, não quiser grossa nem nada, tá?

      Camilla

      Responder
    • Concordo com a Camilla, mas chances não são nem zero, não pra baixo de zero. Estrangeiro não pode nem sequer participar do processo seletivo em medicina. Não entra em pronto.

      Responder
  12. Ola Diego,
    Parabens pela alta qualidade das informacoes. Me desculpe pelo Portugues sem acento, mas meu computador nao esta configurado para o Portugues.
    Bem, queria saber algumas coisas. Sou formada em Enfermagem no Brasil (4 anos), fiz um MBA na Fundacao Getulio Vargas (2 anos), Depois disso, fui morar na Argentina e estudei Medicina por 1 ano e 9 meses em uma escola publica federal la, al final de tudo e atualmente me casei com um americano e moro aqui em Nova York, tranquei a faculdade de medicina e vim pra ca. Tenho um curriculo amplo com servicos de voluntariado em hospital, artigos publicados em eventos cinetificos e ate mesmo em uma revista regional, atividades extracurriculares como participacao em pesquisa cientifica, etc. Fiz conservatorio de canto e violao, falo treis idiomas. Enfim, minha pergunta e: Quais seriam minhas chances de entrar em uma universidade de medicina aqui? qual seria o caminho correto e mais curto que devo tomar? quais as unversidades mais amigaveis com alunos estrangeiros? Desde ja agradeco sua atencao. Sucesso e grande Abraco.

    Responder
    • Olá Amis,

      Seu histórico é muito rico, mas é apenas parte da equação para a “med school”. Em primeiro lugar, você não é mais considerada “estrangeira”. Estrangeiro é a pessoa que não tem autorização para viver nos EUA e não tem como pegar financiamento estudantil. Se você tem social security, você não entra na “fila de estrangeiros”.

      Tem sempre o viés do preconceito contra a pessoa em si (sotaque, aparência não Americana, se for o caso), mas isso é difícil de prever de instituição para instituição. Eu acredito que esse tipo de preconceito siga o mesmo padrão observado na sociedade em geral, o Sul sendo o pior lugar possível para estrangeiros, seguido do meio-oeste e do Norte (Montana, North Dakota). As costas (Leste e Oeste) são conhecidas por serem bem abertas a qualquer cultura, então em NY, não vejo você enfrentando muito preconceito por não ter nascido nos EUA, contudo, a entrada na med school é ampla e os alunos aplicam em todo o país. Nesse caso, mantenha isso em mente.

      Você teria que começar o mais rápido possível a cursar as matérias premed, que são matérias de nível superior (biologia, química, física, matemática, cálculo, psicologia, bioquímica e sociologia) que são cobradas no MCAT (o “ENEM” da medicina) e constituem sua média de ciências (suas notas nas matérias exigidas em medicina). Mesmo que você tenha feito alguma dessas matérias antes, com exceção de matemática, psicologia e sociologia, todas as outras precisam ser feitas nos EUA.

      Premed é um curso disponível em muitas faculdades que também oferecem medicina e você precisa se candidatar para entrar. Não é muito fácil conseguir vaga, mas com seu histórico, acredito que você não tenha problemas. É importante mencionar que você não deve fazer as matérias premed em Community Colleges, pois a maioria das faculdades de medicina não aceita notas dessas instituições.

      Nesse meio tempo em que você estará cursando essas matérias, você precisará construir um currículo de atividades (não necessariamente voluntariado) para mostrar que você se deu o trabalho de observar de perto como é a vida de um médico nos EUA. Isso é super importante e é o maior obstáculo para estrageiros que chegam aqui com um currículo já montado e não querem ir trabalhar por salário mínimo ou fazer voluntariado. Os Americanos são muito bairristas, como você deve ter percebido já, e há um consenso entre os comitês de admissão de que o aluno precisa vivenciar um pouco da vida médica dentro dos EUA para que ele tenha ideia do que vem pela frente e tenha certeza de que é isso mesmo o que ele quer, mesmo que já tenha participado da vida médica antes em outro país, como é o seu caso.

      As atividades mais recomendadas para construir esse currículo são scribe (documentarista) que é o profissional que segue os médicos documentando as consultas (treinamento rápido e pagamento é em torno de $12 a $14 por hora, mas pode ser feito só 1 ou 2 vezes por semana se você tem outra atividade profissional.) CNA (assistente de enfermeira) é outra opção muito usada. Como você já é enfermeira, acredito que você terá muita facilidade para pegar esse tipo de trabalho. Você pode também validar seu diploma de enfermeira e trabalhar diretamente como uma, mas isso tomaria muito tempo e não te daria tempo para fazer as matérias premed e ainda construir um currículo. Só ser enfermeira nos EUA não adiantaria para entrar em medicina, por isso trabalhar full time nessa área não é indicado. Muitos alunos premed fazem EMT também, que é atendimento de emergência e em ambulâncias. Todas essas atividades que eu citei são remuneradas e exigem treinamento, porém curto. Tutoring também é uma boa atividade remunerada (e paga muito bem) para alunos premed que já estão avançados e podem ensinar as matérias premed ou ensinar aulas particulares para alunos de ensino médio.

      Outras atividades não remuneradas, porém muito importantes na construção do currículo são shadowing (seguir de perto um médico por alguns dias ou meses, observando bem de perto todas as suas atividades, mas sem exercer nenhuma atividade), trabalho voluntário como em asilos, hospitais, “fund raisers”, etc.

      No final de todo esse processo (que levará uns 3 anos se você correr), você precisará prestar o MCAT. Como você fez faculdade no exterior, sua nota no MCAT precisa ser mais do que excelente, assim como seu desempenho nas matérias premed. Suas notas em sua faculdade original também contam, mas não tanto quanto se você tivesse feito a faculdade aqui e o curso fosse mais recente.

      O fórum do site studentdoctor.net é altamente indicado para investigar esse processo de entrada e dar uma espiada na vida de um estudante de medicina, assim como alunos residentes e outros já formados, praticando a profissão. Você pode também fazer perguntas por lá.

      Infelizmente, seu tempo na faculdade de medicina na Argentina não vale nada além de uma boa história para contar para o entrevistador da medical school.

      Abraços,

      Diego

      Responder
  13. Seguindo com minha pergunta Diego.
    Outra opcao seria tentar validar meu curso de enfermagem aqui, e depois apresentar para a escola de medicina?
    Ou se eu ativar minha matricula de medicina na Argentina, eu poderia fazer o step 1? e se eu aprovar nesse step 1, conseguiria algo de vantagem no processo?

    muito obrigada pela atencao.

    Responder
  14. Acabei de ler sua resposta e foi muita clara com relacao ao processo. Gostaria de saber sua opiniao sobre 3 coisas;
    1. Voce acha que eu poderia validar meu diploma de enfermagem aqui, E enquanto estou esperando a validacao do diploma, poderia fazer o pre med. ou seja nao necessitaria esperar que a validacao termine para eu comecar fazer o pre med.? Voce acha que mesmo com meu diploma validado aqui com as notas do GPA, seria possivel algum preconceito ainda que minha nota no GPA fosse acima de 3.5?
    Nessa situacao eu gostaria de saber meu GPA, voce havia falando de uma calculadora, poderia me dizer o site e nome por favor? Esta seria uma opcao boa para mim?

    2. OU voce acha melhor eu comecar um college aqui? Se por exemplo minhas notas da Universidade de enfermagem nao estiverem acima 3.5? Ou realmente voce indica o pre med?

    3. Com relacao a Medicina na Argentina, eu vi que estudantes podem fazer a prova o STEP 1. Se eu estivesse matriculada na escola de medicina la na Argentina e fizesse ostep 1 e aprovasse, isso me ajudaria avancar no processo?

    4. Tenho um MBA em que estudei matematica e na Universidade de Enfermagem no Brasil e Medicina na Argentina eu estudei bioquimica, biologia, sociologia, psicologia, seria possivel eu conseguir nao fazer essas materias no College ou no pre-med caso eu apresente documentacao?

    rsrsrs, acho que essas sao minhas duvidas para comecar a agir. Abracos e mais uma vez obrigada

    Responder
    • Olá Amis,

      1. Sim, você pode validar seu diploma de enfermagem. Isso não é só bem fácil, como há uma carêcia de profissionais nessa área e você poderia conseguir facilmente atuar. Contudo, é um “trade-off”, como dizem os Americanos. Ao atuar como enfermeira, isso consumiria bastante tempo que você precisaria estar dedicando para fazer o premed e montar seu currículo de atividades extra-curriculares. Mas como você poderá ver ao pesquisar sobre esse assunto, muitos alunos fazem exatamente isso e trabalham o dia todo, estudanto à noite e fazendo trabalhos voluntários, scribing e shadowing no final de semana ou nos days-off.

      2. O premed não é opcional, mas ele não é precisa ser um curso em si. O curso premed é só para aqueles alunos que só precisam das notas em ciências para poder aplicar para a med school. Pode ser o seu caso se suas notas nas matérias que você pode solicitar equivalência foram realmente muito boas. As med schools gostam de ver no mínimo 2 anos de educação nos EUA de aplicantes que estudaram inicialmente no exterior, então algumas matérias, principalmente as mais importantes (biologia, química e física) precisam ser feitas aqui. Contando que você poderia solicitar equivalência de várias matérias, até seria bom pensar em fazer outro college. Como você está em NY, procure informações sobre a Columbia General Studies, que a divisão da Columbia University que aceita alunos “não tradicionais” como seria o seu caso, e eles oferecem um “fast track” para alunos premed que desejam outro bacharel antes de aplicarem para a escola de medicina.

      3. O STEP 1 é extremamente difícil. Eu não recomendaria simplesmente prestar esse exame sem estar matriculada já em medicina (novamente, seu histórico na Argentina não vai te ajudar). Uma tentativa frustrada poderia marcar você para sempre com o USMLE e não há necessidade nenhuma de prestar esse exame agora. Isso não a ajudaria a entrar em medicina, nem economizaria etapas caso você obtivesse uma boa nota.

      4. Sim, como respondi na questão 2, você pode cortar matemática e cálculo, psicologia e sociologia. Bioquímica pela enfermagem talvez seja possível também. Os únicos que você realmente precisa fazer aqui nos EUA são 1 ano de biologia, 1 ano de química geral, 1 ano de química orgânica e 1 ano de física (todas com seus respectivos laboratórios). Mas você precisará fazer 1 ano de inglês (expository writing) pois sua educação não é baseada nos EUA. Esse inglês não é do tipo curso para estrangeiros, mas matéria de college mesmo.

      Algumas outras matérias não obrigatórias podem enriquecer sua aplicação. Veja esse arquivo PDF. Ele mostra todos os pré-requisitos, e ainda as sugestões de matérias adicionais que podem fortalecer seu currículo: https://ppa.byu.edu/sites/ppa.byu.edu/files/New%20PPA%20Website%20Content/Prehealth/Handouts/Premedical%20Handouts/Premedical%20Coursework%20-%20New%20MCAT.pdf

      Abraços,

      Diego

      Responder
  15. Muito obrigada, estou muito agradecida por poder receber informacoes tao valiosas, como eu poderia ajudar seu site Diego?
    Eu entendi tudo que voce explicou Diego, vejo que tengo um longo caminho pela frente. rs! Me sinto mmm, recomecando tudoo de novo. Mas bem, ainda vou te incomodar um pouco com minhas perguntas.

    Diego, No caso de eu optar por fazer o Collegue, a sequencia seria entao me aplicar para o colegue obvio nesse caso vou verificar com a Columbia, como vc disse. Entao minha pergunta e: A propria Universidade avalia meus grados de Enfermagem e MBA e verifica se eu poderia ter a equivalencia das materias? OU eu teria que enviar meus documentos para uma empresa privada para validar meu titulo primeiro e somente depois apresentar para o Collegue e pedir a equivalencia? Nesse caso os orgaos com autonomia para fazer a validacao seriam?

    Responder
    • Olá Amis,

      Suas perguntas enriquecem muito o site, pois são bem detalhadas e outros leitores podem ter as mesmas dúvidas, então você já está ajudando!

      Para que você possa aplicar para o college, você precisa ter seus diplomas e históricos traduzidos e autenticados por alguma instituição autorizada por cada univerdade em que você está interessada. Não pode ser qualquer uma. Harvard, por exemplo, só aceita documentos traduzidos por uma única empresa. Eu não tenho informações sobre a Columbia, mas isso deve constar no site deles ou entrando em contato por email você descobre.

      Depois de ter sido aceita é que você pode comerçar a tentar eliminar algumas matérias por equivalência, mas não é bom mencionar isso na entrevista, pois as instituições sempre têm preferência por alunos que vão pagar pelo maior número de matérias possível. Se eles souberem que você tem intenção de cortar caminho, arrisca você não conseguir entrar. Então depois de já estar cursando o college, como seu histórico já estará traduzido e em posse do departamento responsável, você faz o procedimento de equivalência. Mais uma vez, em cada escola é diferente.

      Abraços,

      Diego

      Responder
  16. Na verdade seu site e que enriquece o meu pequeno conhecimento sobre esse processo. Tenho lido muito a respeito do tema, mas sao poucos sites que falam a verdade sobre o processo, e quando falam nao tem claridade nas informacoes. Diferente do seu site, que tem precisao nas informacoes. Mais uma vez Parabens, admiro seu trabalho!
    Penso que tenho as informacoes necessarias para comecar a agir, depois de ler aqui, sobre o assunto creio que posso comecar no caminho certo.!!E sobre qualquer duvida agora sei onde posso ter informacao valida e precisa. Sucesso e Abracos.

    Responder
  17. Olá Diego!
    Poderia postar sobre os acessos para o College?
    Quais os critérios para admissão, o s valores, os mais indicados para estrangeiros que querem medicina?

    Responder
  18. Olá Caroline,

    Eu não teria tempo para fazer algo tão detalhado. Acho que isso é coisa que cada aluno interessado precisa fazer por conta, verificando o site de cada instituição em que está interessado, afinal de contas, todas essas informações que você deseja são plenamente explicadas nos respectivos sites de cada escola. Quanto à medicina, medicina não é “college”, como no Brasil. A pessoa interessada em medicina precisa ter um diploma de bacharel (college) primeiro para depois poder se candidatar a uma vaga, então é uma situação completamente diferente.

    Abraços,

    Diego

    Responder
  19. Boa tarde Diego? Tudo bem? Olha eu por aqui outra vez! rs
    Queria te pergunta algo, talvez estupido, mas…vamos la. Estive fazendo uma busca na internet sobre como converte “Carga horaria” digo “Horas aulas” em “Creditos”. Vi muita coisa que explica que por exemplo 1 credito equivale a 15 horas aulas, etc. Mas nao estou convencida completamente sobre isso. Gostaria de te perguntar:
    Voce saberia me dizer um site confiavel onde eu poderia estar verificando como fazer essa conversao corretamente, por exemplo tive 80 horas aula de anatomia, mas eu sei que isso em creditos e outra coisa. E se ao converter esse valor eu encontrar um valor quebrado, ou seja nao completo. Queria fazer essa conversao e depois encontrar meu GPA.

    A razao e simples, rs!!! Se vejo que meu GPA e alto, optaria por TENTAR entrar no PREMED, SENAO, optaria TENTAR (rsrs) entrar no COLLEGE. Queria ter uma ideia mais o menos, claro que terei que traduzir o diploma e fazer todo o processo burocratico. Mas nesse processo de investigacao gostaria de ter uma ideia sobre meu GPA, mas para isso tenho que fazer a conversao primeiro.
    Mais uma vez desculpe pelo Portugues sem acento, computador nao configurado para o portugues.

    Espero que tenha uma excelente semana.
    Abraco
    Amis

    Responder
    • Olá Amis,

      Os créditos não estão relacionados às horas-aulas. Quem calcula seus créditos é a empresa que traduz o histórico, mas no final das contas, se os créditos vão valer alguma coisa ou não depende da interpretação individual de cada escola. Para calcular seu GPA, o que importa mais é sua nota final em cada curso (aula). Na calculadora que eu indico abaixo, as horas-aulas são requisitadas apenas para determinação de validade (um curso de 5 horas não vale nada academicamente) e estruturação visual dos resultados. Vou te dar um exemplo. Na minha faculdade de administração, quando eu comecei as matérias eram anuais, depois passaram a ser semestrais e no último ano eram bimestrais. A quantidade de horas-aulas das matérias bimestrais eram consideravelmente menores do que das matérias que eu fiz no início, mas a quantidade de créditos obtidos foi a mesma. Crédito é uma questão de equivalência. Por exemplo, em Harvard, cada matéria vale 4 créditos (em média). Em um community college na mesma região cada matéria vale 6 créditos, mas se um aluno passar desse community college para Harvard, ele não vai ter mais créditos só porque a instituição dele contava de 6 em 6 ao invés de 4 em 4. Simplesmente haverá uma equivalência e cada matéria de 6 créditos passa a valer 4.

      O GPA você calcula com base na sua nota final em cada matéria, o que deve estar indicado no seu histórico. Por exemplo, se sua média geral em anatomia foi 9.0 no sistema brasileiro, isso é equivalente a um A- no sistema Americano. Em uma calculadora de GPA, você deve ser capaz de inserir as letras para cada nota média e obter seu GPA. Tudo isso independe de horas-aulas ou de créditos. Esse cálculo de créditos e horas-aulas é feito somente na autenticação do diploma e histórico para que seja determinado que cada matéria cursada equivale ao conteúdo esperado. Isso serve apenas para eliminar matérias que foram cursadas de forma insatisfatória, como em um curso de final de semana. Existe um número mínimo de horas necessárias para que uma matéria seja considerada como crédito válido.

      Essa calculadora de GPA é ótima: http://www.foreigncredits.com/Resources/GPA-Calculator/

      Você precisa antes encontrar uma tabela para conversão das notas em sistema de letras e são essas letras que você colocará na calculadora.

      Abraços,

      Diego

      Responder
  20. Obrigada Diego, esclareceu muita coisa na minha mente! vou verificar essa calculadora e pesquisar sobre a tabela para conversão das notas em sistema de letras. Entendo que a Escola avalia que os creditos ou GPA e valido ou nao, ainda mais porque se trata de escolas no Brasil que eles aqui nao tem como avaliar ne. Um “A” em Anatomia por exemplo tirado em uma escola no Brasil talvez ou creio eu nao tem o mesmo peso que um “A” tirado em uma escola aqui ne. E me parece que eles tem isso muito em claro ne. Nesse caso muitos de nos que fizemos College no Brasil saimos perdendo TAMBEM nesse quesito.

    Abraco
    Amis

    Responder
    • Olá Amis,

      Essa comparação é verdadeira, mas não é relevante. Um A é um A. São os créditos, ou melhor, a forma como cada escola calcula os créditos, que variam. As notas que constam no seu histórico formam um GPA objetivo, independente e válido para qualquer escola, desde que o curso que você tenha feito tenha tido duração de no mínimo 4 anos (o que é equivalente ao college Americano).

      Abraços,

      Diego

      Responder
  21. Olá ! Eu tentei processar todas as informações, então vamos lá ! Eu estou no ensino médio aqui no Brasil e gostaria de realizar o curso de medicina nos Estados Unidos. Primeiro eu teria que entrar em algum “college” ( O college inclui qualquer faculdade, ex: Stanford, Yale, Harvard, etc ?? / Se eu desejar realizar o HMS eu teria que necessariamente fazer o college em Harvard, ou pode ser em qualquer outra universidade? ), e no college eu teria que escolher algum curso que envolvesse todas as 8 matérias ( então se eu passasse em uma faculdade no exterior eu teria que escolher, por exemplo biologia? ) Aí, somente depois eu irei realizar o MCAT, que ira decidir se eu passei diretamente no curso de medicina que seria a “pós-graduação” ?( Além de todos os outros requisitos, carta, histórico…)
    * Mas se eu passei no college com bolsa de mérito/financeira ela só ira servir para o college,e depois eu que irei arcar com as despesas ?
    * Programas como o Ciência sem Fronteira, e outros, é o mesmo processo ou é diferente? Como funciona neste caso ?
    * Para realizar medicina em outros países ( Itália, França, Alemanha…) é mais fácil ou mais complicado ?
    * Antes de entrar no college, eu teria que realizar o SAT I, II e etc, né?
    **** Eu estou correndo atrás da minha dupla cidadania que seria a Italiana. Se eu conseguir ter a cidadania brasileira e italiana, como ficaria minha situação?

    Responder
    • Olá Maria Eduarda,

      Sem Green Card ou cidadania Americana, você teria que pagar seu curso de medicina à vista (pode chegar a 250.000 dólares, incluindo as despesas de vida) ou precisaria de um “sponsor”, que é um fiador que assume o financiamento, mas isso só funcionaria se você tiver parentes morando nos EUA que tenham Green Card ou cidadania. Sua cidadania Italiana não ajuda em absolutamente nada sua entrada em qualquer instituição nos EUA, mas pode obviamente, facilitar sua entrada em uma instituição Européia.

      Vamos lá então com as questões mais específicas:

      – College é igual a “nossa faculdade” no Brasil. Se você entrar em qualquer faculdade no Brasil, digamos, engenharia mecânica ou letras, ao se formar você terá um diploma de “college”, também chamado de “undergraduate degree”.

      Para entrar em medicina, você precisa ter um curso superior (college, faculdade, undergraduate) em qualquer coisa e eu já vi gente das mais variadas áreas entrando em medicina, de letras à antropologia. O que quis dizer com relação às matérias na escolha do curso é que para medicina o aluno precisa montar um currículo de matérias específicas cursadas (1 ano de química geral (+ lab.) , 1 ano de química orgânica (+ lab.), 1 ano de biologia (+lab), 1 semestre de matemática, 1 semestre de cálculo, 1 ano de inglês (não vale curso de inglês, tem que ser inglês de college Americano mesmo), 1 ano de física (+ lab.), 1 semestre de sociologia, 1 semestre de psicologia e 1 semestre de bioquímica). “Lab.” evidentemente é aula de laboratório. Como o aluno precisa cursar todas essas matérias, quem já sabe no ensino médio (high school) que deseja fazer medicina, geralmente escolhe um curso que seja compatível e que ofereça a maior quantidade de matérias chamadas “premed”. Os que fazem uma faculdade que não contém alguma ou todas essas matérias, precisam cursá-las individualmente ou fazer um curso chamado premed após terminar a faculdade (college). A maioria faz premed, mesmo já tendo feito essas matérias antes para melhorar as notas.

      – Você não precisa fazer college nos EUA obrigatoriamente. É ideal, mas não é obrigatório. Estudar no Harvard College não é, de forma alguma, porta de entrada para a Harvard Medical School.

      – Não existem bolsas para a faculdade de medicina, nem sequer para os Americanos. Se você não puder pagar e não tiver Green Card, cidadania Americana ou parente que tenha para ser fiador e pegar um empréstimo, você não pode cursar. A grande maioria das faculdades de medicina nem sequer chamam para entrevista alunos estrangeiros que não tenham provado que têm condições ou de pagar à vista ou de pegar financiamento. Seria um desperdício de recursos por parte deles entrevistar candidatos que não poderia pagar pelo curso se fossem aceitos e eles realmente levam isso à sério.

      – Bolsas para estrangeiros no college também são raras. Geralmente atletas, celebridades internacionais ou pessoas que possuam algum destaque muito significativo conseguem bolsas sendo estrangeiros.

      – Se você não tem condições de pagar pelo college, mude seu foco já para a Europa, pois não há como contornar esse problema, a não ser que você viajasse para os EUA como turista e se casasse com um Americano para ganhar um Green Card! Se isso não é uma opção, é melhor não ficar sonhando com faculdade nos EUA. Muitas faculdades na Europa são gratuitas e você terá direito de estudar nelas.

      – Entrar em medicina na Europa é MUITO mais fácil, não tem nem comparação. O processo de seleção de medicina aqui nos EUA é completamente insano! O cara tem que trabalhar durante anos focado nesse objetivo de entrar em medicina, tirando notas perfeitas (um mero “8” pode arruinar as chances da pessoa entrar em medicina), fazendo trabalho voluntário enquanto estuda para sempre tirar 10 em matérias extremamente difíceis como química orgânica e física (de nível superior, o que você aprende nessas matérias no ensino médio nem chega perto do conhecimento necessário para se dar bem no premed). Além do mais, medicina na Europa é como no Brasil, você entra direto após o ensino médio, não é necessário já ter um curso superior como ocorre nos EUA. A maioria também usa sistema de vestibular, o que é bem mais democrático e simples.

      – Você fala sobre entrar em Stanford, Harvard ou Yale como se fosse uma escolha sua! Não é assim! Alunos estrangeiros têm muito pouca chance de entrar nessas instituições e quando isso acontece, geralmente são alunos que já obtiveram extremo destaque, tanto na escola com notas perfeitas quanto fora dela como ganhando medalhas olímpicas ou com histórias de vida realmente impressionantes.

      Eu recomendo que você estude melhor suas possiblidades para manter os pés no chão e não ficar se iludindo com coisas que estão fora do seu alcance. Não quero te puxar pra baixo, não! Mas é preciso manter os pés no chão para planejar e manter o foco no longo prazo para que você chegue ao seu objetivo.

      Abraços,

      Diego

      Responder
    • Eu sou só leitor aqui do site, mas posso confirmar o que o Diego escreveu e ainda acrescentar mais algumas coisas que ele não respondeu.

      Realmente, com cidadania Italiana, você não tem nenhum benefício com relação às escolas Norte Americanas. Porém, isso abre as portas para você estudar na Europa, onde é muito mais fácil entrar em medicina (ou qualquer outro curso, comparando com os EUA).

      Programas como ciências sem fronteiras serve apenas para o aluno cursar um período curto (geralmente 6 meses ou 1 ano) do seu próprio curso no Brasil em alguma escola no exterior. Por exemplo, a USP tem um convênio com Harvard para enviar alunos de intercâmbio no curso de medicina. Os alunos podem passar 1 ano estudando/fazendo pesquisa em Harvard, mas depois precisam voltar para o Brasil para completar o curso. Esse 1 ano não conta para a faculdade e o estudante precisa ainda preencher seus 6 anos de medicina no Brasil. O intercâmbio não é parte da faculdade e não vale para créditos ou diminuir a carga necessária para se formar. Da mesma forma, alunos de mestrado e doutorado podem fazer esses intercâmbios para (geralmente) fazer pesquisa em alguma instituição no exterior, mas logo depois são obrigados a voltar para o Brasil e “retribuir” o tempo no exterior com dedicação à faculdade no Brasil.

      Você jamais poderia fazer um curso inteiro, ainda mais medicina, no exterior, quer seja Europa ou EUA, através de qualquer programa de bolsas no Brasil. Bolsas, assim como ocorre no Brasil, estão disponíveis somente para cidadãos do próprio país. Um estrangeiro não pode vir para o Brasil e estudar com bolsa, da mesma forma, você não pode ir para os EUA e ganhar bolsa. Como o Diego explicou, instituições privadas de vez em quando oferecem bolsas para alunos extraordinários, mesmo que sejam estrangeiros, mas isso é raro.

      Se você quer fazer college nos EUA (que não pode ser medicina, como já foi explicado), você precisa definir como vai pagar, pois nenhum tipo de ajuda financeira estará disponível em sua situação. Se você não tem como pagar, foque-se em fazer essa faculdade na Europa.

      Minha dica pra você é a seguinte: como ainda vai demorar um tempo para você obter a cidadania Européia (essas coisas demoram, mesmo que você “corra” com o processo), o ideal é que você faça faculdade no Brasil mesmo. Com seu diploma de bacharel e sua cidadania Européia você poderá fazer mestrado ou pós graduação na Europa. Se seu foco é mesmo medicina, faça no Brasil. Se você sonha em morar nos EUA, você pode futuramente fazer residência nos EUA e alguns testes que são necessários para validar o diploma (da mesma forma como ocorre no Brasil). Medicina é um curso extremamente intenso que irá consumir todo o seu tempo e não permitirá com que você possa trabalhar para se sustentar. O ideal é que você passe por esse processo estando dentro da casa dos seus pais, onde você não precisa se preocupar com dinheiro ou tomar conta de si mesma (limpar casa, lavar roupa, ir no supermercado, etc.).

      Abraços e boa sorte com seus sonhos!

      Carlos

      Responder
      • Olá Carlos !! Obrigado. Realmente suas informações e opiniões ajudaram muito !
        Como você disse, realmente demora bastante para obter a cidadania, e eu disse no comentário abaixo ( eu não tinha visto seu post quando publiquei ) eu acho que prefiro a Europa mesmo. Eu já fui para Europa , e como parte da minha família é de lá eu tenho um “apego” maior e terei mais facilidades, eu acho.
        A minha prima conseguiu uma bolsa e vai passar um ano lá nos EUA, mas eu se eu for para o exterior é para ficar. Por isso eu estou pesquisando bastante e incomodando as pessoas com perguntas ( me desculpem ahshs ) Não é qualquer escolha, então eu e meus pais estamos correndo atrás logo cedo.
        A sua dica realmente é muito boa, e é o que a maioria das pessoas estão me aconselhando. Fazer a faculdade aqui é muito mais vantajoso em alguns aspectos, principalmente em medicina que é um curso extremamente intenso, como tu disse. Mas é bem complicado para mim essa escolha, pois eu gostaria de já começar lá.
        Eu agradeço muito a opinião, conselhos e informações de vocês, pois para mim este está sendo um período muito importante e eu quero tomar as decisões certas com calma e ajuda da minha família.

        Muitooooo Obrigado ! Abraços 😀

        Responder
        • Quando a gente é jovem, a gente sofre de um mal terrível, falta de perspectiva e pressa! Tudo tem que ser pra ontem, como se a vida fosse acabar aos 30!

          Pra que tanta pressa?

          Eu sei que você pode vir com milhares de respostas, mas a realidade é que não há qualquer razão REAL para você sair do país AGORA (ou no futuro próximo).

          Eu acho que qualquer dica que a gente possa te dar vai depender de um fator muito importante que você ainda não mencionou: qual a sua condição financeira? Seus pais têm condições de arcar com as despesas em Euros de você estudar na Europa e se manter em um outro país sem precisar trabalhar?

          Isso é importante, porque ao contrário do Brasil, faculdade nos EUA e na Europa é período integral. Não dá pra estudar e trabalhar. Eu não quero jogar um balde de água fria nos seus planos, mas se seus pais não têm condições financeiras de te manter em outro país, não adianta ficar sonhando, é melhor fazer a faculdade no Brasil mesmo e depois ir para onde você quiser trabalhar e expandir a sua educação. Residência é uma opção, já que residência médica não é um “curso”, mas sim um “emprego”. Você faria medicina no Brasil e iria fazer residência em outro país, aí sem ter que se preocupar com essa questão financeira.

          Abraços,

          Carlos

          Responder
  22. Olá novamente ! Desculpa incomodar :3

    Obrigado pelas informações, deu para esclarecer bastante as minhas maiores dúvidas. Então eu estava pensando em realizar alguma faculdade no exterior pois é basicamente a minha vontade. Eu acho que eu tenho uma estrutura boa ,apoio e força de vontade para conseguir o que eu quero, e nunca desistir.
    No começo eu fiquei interessada em faculdades nos EUA pois todo mundo comenta, e porque realmente a maioria é excelente! Mas eu acho que a minha vontade realmente é estudar na Europa. Só que assim, o motivo que torna a facilidade de entrar em uma faculdade na Europa , diminui a qualidade ? Eu particularmente acho que não, pois você tem ótimas faculdades super conhecidas pela qualidade de ensino. Mas quando comparamos uma faculdade da Europa com uma dos EUA tem alguma diferença ? ( de ensino, qualidade, mercado de trabalho…) Por que todo mundo, basicamente só fala de EUA, eu acho que ter um ensino ótimo é muito bom, mas como algumas pessoas dizem: quem faz a faculdade é o aluno. Eu desejo passar em uma faculdade que tenha ótima qualidade no meu curso, e que não seja necessariamente de renome.
    Eu não sei se o que eu vou falar é besteira, mas já escutei dizerem que é possível encontrar uma pessoa mais esforçada e dedicada em uma faculdade “normal” do que em uma “famosa”, eu acho que acredito nisso. ( faculdades em geral, não só EUA, etc..) Isso é algo que desanima bastante gente por aí sabe. Se você não está em uma faculdade excelente você não irá ser um profissional excelente. Eu acho que não é bem por aí.
    Se você puder recomendar algum site sobre como estudar na Europa, ou você mesmo fazer um post eu agradeceria, pois tu é bem realista e sabe como explicar( e é difícil encontrar sites assim) haha Obrigado e desculpe se falei alguma besteira !

    Ótimo site !!!! Abraços 🙂

    Responder
    • Olá Maria Eduarda,

      Não se acanhe para fazer perguntas, mesmo que você as considere “besteira”, nunca é. Outras pessoas também têm as mesmas dúvidas que você tem e elas podem encontrar as respostas que procuram as ler os comentários aqui no blog!

      Não vá muito atrás do que as pessoas “dizem”. A grande maioria das pessoas não sabe muita coisa sobre instituições de ensino no exterior e seus comentários refletem mitos e desconhecimento. Há muitas escolas excelentes nos EUA, mas a Europa não fica pra trás, não! Da mesma forma, há péssimas escolas nos EUA e também na Europa, é preciso saber escolher.

      Essa escolha, contudo, depende muito do que **exatamente** você pretende alcançar na vida, pois ao contrário do que o “bom senso” afirma, a instituição em que você estuda define SIM as oportunidades que você terá depois de formada. Se você fizer uma faculdadezinha de quinta categoria, não importa o quão esforçada ou boa aluna você seja, suas oportunidades serão sempre limitadas. Você pode até encontrar uma pessoa esforçada em uma faculdade sem gabarito, mas e daí? O que isso tem a ver com você? É isso o que você tem que perguntar a si mesma. Como o gabarito de uma instituição vai impactar os seus próprios objetivos? No meu caso, eu trabalhei em Harvard e em Oxford. Eu JAMAIS teria tido a oportunidade de trabalhar nessas instituições se eu não tivesse estudado em Harvard, por mais esforçado que eu fosse, entende? Até um certo ponto, o pedigree (ou falta dele) da escola que você faz, te persegue pelo resto da vida. Não é uma questão da pessoa ser ou não um profissional excelente, mas sim das oportunidades que estão abertas para ela porque ela fez uma faculdade de destaque, ao passo que para a pessoa que fez uma faculdade de menor destaque, muitas portas permanecem fechadas.

      Em medicina, contudo, se é isso mesmo o que você deseja fazer, essa diferença diminui consideravelmente, pois como médica você pode simplesmente abrir seu consultório e praticar sua especialidade, quer você tenha se formado na USP ou em uma escola de menor prestígio. Mais uma vez, contudo, depende do que você quer.

      Por que você quer fazer medicina?

      Quais são seus planos para sua vida no longo prazo?

      Esse tipo de resposta poderá ajudá-la a definir com mais clareza que tipo de instituição de ensino proporcionará a educação e o gabarito que você precisa para alcançar os seus objetivos. Em qualquer país você pode encontrar aquelas faculdadezinhas de quinta categoria que existem só para fazer dinheiro para seus donos, como uma empresa convencional. Ao escolher uma faculdade na Europa, você precisa investigar muito bem a reputação da mesma, para não cair na armadilha de achar que é boa “só porque é na Europa”. O mesmo é válido para os EUA.

      Não conheço nenhum site especializado em escolas na Europa para te indicar, mas isso não deve ser difícil encontrar pelo Google. Prefira sites em inglês, de preferência, os que possuem forum de discussão.

      Abraços,

      Diego

      Responder
    • Eu to me metendo aqui nessa conversa, mas só queria reforçar o que o Diego disse. O pedigree da faculdade que você faz é muito importante. Eu trabalho no RH da McKinsey Consultoria que é praticamente a maior empresa de consultoria do mundo e muita gente quer trabalhar aqui. Currículos de gente que fez faculdade de segunda linha vão direto pro lixo. Eles não querem saber se a pessoa é “bom profissional” ou não, se não fez escola de primeira, não entra na McKinsey, ponto final. Google é a mesma coisa, só contrata formados em Stanford, Harvard, MIT e cia. A filosofia é que se a pessoa fosse “boa” mesmo ela teria conseguido entrar em uma escola de ponta, se não conseguiu, é porque há uma razão: ela não é tão boa! É a mesma coisa no Brasil, se a pessoa não conseguiu entrar em uma federal, é porque ela não tem tanto gabarito. A faculdade que a pessoa fez já mostra o potencial dela.

      Responder
  23. Obrigada, li todo o artigo (tinha perguntado sobre odonto na matéria dos SAT’s). Muito esclarecedor, mas só queria saber se entrar em odonto e tão difícil quanto você diz ser difícil entrar em medicina afinal? Ou tem a possibilidade de ser menos burocrático?

    Responder
    • Olá Natália,

      A burocracia é a mesma. Você precisa dos mesmos elementos: GPA muito alto, matérias científicas cursadas em curso superior (de preferência em instituição nos EUA) com notas praticamente perfeitas (não tantas quanto medicina, mas ainda assim é praticamente um “premed” com biologia, física e química e seus laboratórios, matemática e cálculo, bioquímica e inglês). Você também precisa de um currículo de trabalho e voluntariado na área odontológica, passará por uma entrevista pessoal com o comitê de admissão de cada instituição e precisará de nota alta no teste de admissão, o DAT (Dental Admission Test). E ainda as cartas de recomendação, o atestado de propósito pessoal (uma redação em que você explica porque quer fazer odonto) e o essay (uma redação sobre a sua vida e a sua trajetória e como sua história lhe trouxe para odontologia). É quase tão difícil quanto entrar em medicina. Muito melhor fazer odonto no Brasil mesmo e se quiser uma educação nos EUA, venha fazer pós, mestrado ou doutorado.

      Abraços,

      Diego

      Responder
        • Olá Natália,

          Nenhum curso ou formação te dá o direito de ficar nos EUA, muito menos de trabalhar, nem mestrado e doutorado. Depois que você termina o curso, se nenhuma empresa Americana te contratar (o que não é garantido), você é obrigada a deixar o país e voltar para o Brasil.

          Abraços,

          Diego

          Responder
          • Sim, mas se eu tiver nacionalidade americana e me formar em odonto no Brazil e fizer especialização, seja mestrado, doutorado nos EUA, eu consigo trabalhar nos EUA tranquilamente?

          • Com nacionalidade Americana, voê não precisa fazer curso nenhum para trabalhar nos EUA, evidentemente. Isso muda tudo e você não mencionou em sua pergunta que era Americana ou tinha a possibilidade de vir a ter cidadania. Se você tem cidadania, você não precisa fazer mestrado ou doutorado para trabalhar no país, basta validar o diploma de odontologia, assim como é feito no Brasil quando a pessoa se forma no exterior. Eu não sei os detalhes desse processo em odonto, você precisa pesquisar.

            Abraços,

            Diego

  24. Vocês já pensaram em fazer medicina aqui na América Latina? Meu irmão faz medicina no Uruguai, na universidade Claeh (http://goo.gl/GTU9ZJ) ele estuda a dois anos no Uruguai e está adorando. Quando ele escolheu estudar no exterior pesquisou várias universidades até se decidir pelo Claeh.

    Responder
  25. Olá tenho 19 anos faço cursinho para medicina no Brasil porém tenho planos para me mudar para os EUA gostaria que vc me orientasse por onde começar o que fazer pois estou um pouco perdida!
    Já faço trabalho voluntário no hospital do câncer infântil em São paulo a 2 anos, e tenho muitos planos para ir para os eua!
    Aguardo sua ajuda

    Responder
    • Rafaella… Não quero ser chato, por favor me perdoe, não quero ofender você, nem nada, mas você me parece muito ingênua com essa história toda… Em primeiro lugar, você não pode simplesmente “se mudar” para os EUA só porque você quer. Não é assim que funciona… Você precisa, em primeiro lugar, ter condições financeiras de se manter lá, você tem? Ou melhor, seus pais tem condições de te bancar lá? (contando que você como estrangeira não pode trabalhar, não pode ter nenhuma fonte de renda no país). Como estrangeira, você também não qualifica para bolsas estudantis, nem financiamento, teria que pagar o estudo em cash e o valor para estrangeiros é mais do que o dobro.

      Entrar em uma faculdade já outra questão e não seria medicina, pois como o cara do blog falou, você ainda não pode fazer medicina nos EUA porque é muito nova e não tem curso superior. Isso sem falar nos inúmeros empecilhos que a própria medicina impõe, a começar pelo fato de que estrangeiros raramente são aceitos neste curso nos EUA. Se você não é Americana nem tem Green Card, você nem deve começar a sonhar em fazer medicina nos EUA pois isso provavelmente não vai acontecer.

      O melhor, como já foi dito em inúmeros comentários aqui, é fazer medicina no Brasil e depois ir fazer residência nos EUA, que é o processo de “validação do diploma” lá. Aí sim você consegue ir “morar lá” como sonha.

      Mas lembre-se, antes de ficar sonhando, faça bastante pesquisa, não fique nessa ingenuidade toda.

      Responder
    • Olá Rafaella,

      Com 19 anos você não tem como entrar em medicina nos EUA. Por favor leia o artigo e demais comentários, pois essa questão é básica. Você precisa primeiro de um curso superior. Medicina nos EUA é pós-graduação.

      Abraços,

      Diego

      Responder
  26. Olá, por favor, você teria alguma indicação de um CONSULTOR para orientar a aplicação para o curso de MEDICINA nos EUA?
    Como foi citado no texto acima, sobre um rapaz que procurou uma consultoria após várias tentativas e não ter sido aprovado ainda…
    Por favor, caso você não tenha esta indicação, você saberia informar ONDE posso procurar indicações para este tipo de serviço??
    Alguém que possa analisar meu histórico, currículo e minhas atividades extracurriculares e avaliar minhas chances de ingressar na faculdade ou me orientar em algo que eu possa complementar.
    Você conhece alguém que possa fazer esse serviço, por favor?

    lhe agradeço desde já.

    Responder
    • Olá Kynha,

      Há consultores aos montes, é só você procurar no Google (em inglês, é claro). Eu, particularmente, não conheço nenhum, mas não é nem um pouco difícil encontrar um profissional bom. Contudo, como já discutimos bastante por aqui, você só tem reais condições de entrar em medicina nos EUA se tiver Green Card ou for cidadã Americana. Vale muito mais a pena entrar em uma Universidade Federal aqui no Brasil (que é, aliás, muito mais fácil do que entrar em medicina nos EUA, além de ser de graça) e depois fazer residência nos EUA se sua intenção é trabalhar no país depois de formada. Ao contratar um consultor, você pode acabar sendo enganada. O consultor não garante, obviamente, que o aluno vai entrar, mas pode dar esperanças na tentativa de que você o contrate, quando a realidade é que as chances de um estrangeiro entre em medicina nos EUA é praticamente zero. Eu em toda a minha carreira, NUNCA conheci um Brasileiro (sem ser naturalizado) que tivesse entrado em medicina aqui.

      Abraços,

      Diego

      Responder
  27. Olá!!!

    Queria saber a sua opinião sobre o meu caso. Me formei há poucos anos em odontologia e atualmente estou fazendo o segundo ano do curso de medicina, ambos no Brasil. Há alguma possibilidade de conseguir continuar meus estudos médicos nos EUA? Ou seria melhor concluir o curso aqui e depois fazer a residência lá? A minha intenção é realmente de viver o resto da vida por lá – e o quanto antes. Tenho 25 anos, curso de 4,5 anos de inglês, consigo me comunicar bem, mas precisarei aprofundar mais os estudos nessa área, sem dúvida. Tenho estágio de alguns anos em hospitais de urgência e emergência e clínicas, minha experiência de trabalho no consultório, produção científica e premiações no currículo. Além de notas muito boas em ambos os cursos. Inclusive recentemente um trabalho da minha autoria ganhou a primeira colocação geral em um congresso e o prêmio será a viagem para os EUA + um curso de anatomia em Orlando no final desse ano. Enfim, não sei isso poderá servir de algo, mas gostaria de saber as suas dicas para o meu caso.

    Muito obrigada!!

    Responder
    • Olá Renata,

      Eu acho que você deve se focar em terminar seu curso no Brasil e depois fazer residência nos EUA. É um plano muito mais realista e com chances de dar certo. Faculdade de medicina não tem como você “aproveitar matérias”, você teria que começar de novo. Além disso, as chances de que você consiga entrar são ínfimas. Você tem um currículo bom, mas você não faz ideia do potencial e currículo do aluno típico aceito em medicina nos EUA. Além disso, você teria que prestar o MCAT, um teste que é infinitamente mais difícil do que o vestibular. Você já passou por isso, já estudou todas aquelas matérias que você nunca mais vai precisar estudar, pra que ter que revisar tudo isso, em um nível ainda mais profundo do que você precisou fazer para passar no vestibular?

      Outro ponto, você disse que estuda com financiamento no Brasil… Como você pagaria o curso de medicina nos EUA que no câmbio de hoje passa de 1 milhão de Reais? Nenhum financiamento estaria disponível para você, que não é cidadã americana, e sendo assim, se numa eventualidade você fosse aceita, você seria obrigada a pagar o curso todo à vista como condição para entrar no curso. Como discutimos várias vezes aqui nesses comentários (e nos comentários do artigo sobre medicina em Harvard), a maioria das faculdade de medicina nos EUA veta completamente alunos estrangeiros, e as que aceitam exigem pagamento integral do curso para aceitação do aluno, justamente porque estrangeiro não consegue financiamento e sendo assim, é um risco para as instituições darem uma vaga para um aluno que pode não conseguir pagar o curso todo.

      Recomendo que você leia aqui no blog o relato de um brasileiro, naturalizado americano, que tentou por 4 anos entrar em medicina nos EUA, estando lá e preenchendo todos os pré-requisitos. Ele era perfeito, mas mesmo assim, não foi aceito em nenhuma escola.

      Agora, terminar seu curso no Brasil e aplicar para residência nos EUA é um plano completamente diferente e realista. Você precisa ir muito bem no seu curso no Brasil, é claro, e ir prestando os testes do USMLE enquanto está na faculdade. Procure no Google por STEPS do USMLE. Você pode prestar esses testes estando no Brasil. Eles são muito difíceis e cobram o conteúdo do curso de medicina em inglês, que você deve começar a estudar desde já. Ao aplicar para a residência, você enviará as notas dos dois primeiros STEPS e fará uma entrevista pessoal, que acredito que hoje em dia, já façam por Skype com alunos que estão fora do país.

      Abraço,

      Diego

      Responder
  28. Esse realmente foi o artigo mais pé no chão que já li sobre medicina no exterior. Parabéns!

    Eu me formei em Ciências Biológicas agora e estou pensando em cursar medicina fora do país por alguns motivos: excelência, eu aproveitaria minha graduação, o tempo seria mais curto, o preço sai quase o mesmo do que no Brasil.
    Meu GPA é 3, TOEFL 113 e ainda não prestei MCAT… Já fiz intercâmbio na universidade da austrália e já fiz trabalho voluntário numa org da áfrica. Faço luta e dança, mas nunca ganhei nenhuma medalha. Já publiquei alguns artigos científicos em revistas internacionais. Pretendo fazer medicina e residência nos EUA ou Canadá (talvez eventualmente no futuro voltar para o Brasi), mas agora lendo tudo o que você escreveu, não sei se isso seria suficiente para ser aceita em uma universidade como Stanford ou Uni of British Columbia (Canadá)… O que você acha?
    Não daria para estudar e fazer o processo de inscrição daqui do Brasil e só ir para os Estados Unidos se for aceita para entrevista?

    Muito obrigada!

    Responder
    • Olá Thais,

      Estrangeiros (sem Green Card) raramente são aceitos na faculdade de medicina. Até mesmo para estrangeiros que possuem residência ou cidadania, ainda assim, o preconceito é muito grande e a probabilidade de ser aceito é mínima (leia o artigo do Marcos aqui neste site sobre a experiência dele em tentar entrar em medicina nos EUA sendo cidadão americano, mas tendo nascido no Brasil). Não é a mesma coisa que fazer algum outro curso, incluindo cursos competitivos como direito.

      O custo não é similar ao Brasil, não! O curso de medicina custa em torno de $250.000 (incluindo custos de vida), quase 1 milhão de Reais no câmbio de hoje. Além disso, um estrangeiro, se aceito, teria que pagar o curso todo à vista.

      Outro ponto, o perfil do aluno que entra em Stanford e outras universidades similares é completamente inacreditável para o nosso padrão. Você tem alguns pontos positivos, mas há muitos outros alunos que também têm o que você tem e muito mais. Medalhas e hobbies são coisas que importam para entrar no college (faculdade inicial). Graduate school, como medicina, exige mais sofisticação no currículo e engajamento direto com a atividade assistencial e médica. GPA 3.0 é extremamente baixo para medicina, isso é o que te prejudica mais. A média dos alunos que entram mesmo nas faculdades meia boca é 3.5. As top como Stanford, você vê GPA 3.8, 3.9.

      Não valeria a pena perder seu tempo estudando para o MCAT, um teste que coloca o vestibular no chinelo. Além disso, acredito que você ainda teria que fazer algumas matérias premed como física (com laboratório), cálculo, sociologia, psicologia, e bioquímica, se não fez na graduação. Isso tomaria 1.5, 2 anos (os 2 anos extras que você está tentando economizar!).

      Acho irreal pensar em fazer medicina no EUA (outros lugares, talvez, mas eu não sou a pessoa mais indicada para te aconselhar, pois só conheço o processo de entrada na med school Americana). Para quem tem gabarito para passar no vestibular no Brasil, o ideal é se focar em fazer uma faculdade pública. Não paga nada e você tem uma educação de primeira. Vai fazendo os 2 primeiros STEPs do USMLE e, depois de formada, você aplica para residência nos EUA.

      Abraços,

      Diego

      Responder
  29. Olá. Ano que vem eu começo o ensino médio e quero cursar college e med school nos EUA. Tenho interesse em Harvard e sei da baixa taxa de aceitamento. Gostaria de uma opinião sincera sobre as chances de entrar na Med School. Obrigada desde já.

    Responder
    • Olá Marina,

      Isso já foi amplamente discutido aqui e no artigo sobre medicina em Harvard. As chances de um estrangeiro entrar em medicina nos EUA, seja Harvard ou qualquer outra faculdade, é praticamente zero – por inúmeros motivos. Não existem bolsas em medicina e o curso custa em torno de 200 mil dólares. Como alunos estrangeiros não tem como conseguir financiamento estudantil (como fazem os Americanos) os comitês de admissão preferem nem sequer considerar esses alunos. Também há grande preconceito da sociedade americana contra médicos estrangeiros ou médicos que venham “se aproveitar” da educação nos EUA para depois voltar aos seus países de origem. A população faz pressão para que as faculdades de medicina não dêem vagas para estrangeiros. Outro motivo é que o rigor de medicina é altíssimo e as faculdades se preocupam em aceitar alunos que tenham comprovado que possuem gabarito para aguentar o tranco. Nesse ponto, alunos estrangeiros também saem perdendo pois os comitês de admissão (particularmente de medicina) consideram a educação em países como Brasil de “baixa qualidade”. Enfim, muito disso é irreal ou injusto, mas infelizmente é assim que funciona. Como já discuti com outros visitantes aqui, o caminho melhor é sempre fazer medicina no Brasil e depois ir para os EUA fazer residência, que á um emprego e paga muito bem (aí a pessoa não precisa se preocupar com subsistência ou vistos).

      Abraços,

      Diego

      Responder
  30. Olá Diego, parabéns pelo blog… o melhor que encontrei, muito informativo.

    Bom, vc já aclarou muita coisa nas outras perguntas. Só uma duvida, na verdade quero mais um conselho seu…

    Para não ter que alongar em detalhes, tenho 34 anos e decidi fazer medicina. Pela idade não quis perder tempo com cursinhos para vestibular e muito menos teria tempo para me preparar para ser aceito em nenhum medical school nos USA. Então vim para Assunção onde curso atualmente o 1ro ano de medicina. Estava querendo transferir para Moscow na Russia, somente pelo fato das aulas serem administradas em ingles e pela faculdade ter mais nome. Mas pelo que eu entendi para os Estados Unidos só existe Estados Unidos, Canada y os “outros países”.

    Sou cidadao americano também e queria muito voltar para “casa” um dia. Digo casa, não por ter nascido nos Estados Unidos mas por minha familia morar lá. Terminei minha High School nos Estados Unidos tambem. Claro que a um bom tempo atrás.

    Enfim, gostaria muito de saber qual seria sua dica para eu estar ja me preparando para tentar essa volta no futuro. Para residencia eles também avaliam toda vida extracurricular do “aplicante”. Trabalhos voluntarios etc etc etc…. Ou só a nota dos Steps no USMLE? E acho que vc já respondeu essa pergunta, mas só pra confirmar. Vc indicaria a transferencia para outra faculdade na Europa ou o melhor caminho seria continuar onde estou e não perder mais tempo. E usar esse ano que perderia para estar sempre me preparando para as provas de Residencia nos Estados Unidos?

    Muito Obrigado, grande abraço.

    Responder
  31. Olá, sou aluna do 2 ano do ensino médio, e faz muito tempo que quero estudar nos Estados Unidos, esse ano confirmei meu desejo de fazer medicina, pelo que eu li no artigo e nos comentários eh realmente muito dificil entrar numa faculdade de medicina nos eua, agora oq eu gostaria de saber eh: eu sou uma boa aluna, não diria que uma aluna nota dez em
    Tudo, mas sou constantemente considerada uma das melhores do meu ano em relação a todas as classes, estou começando a fazer trabalho voluntário com crianças deficientes e animais, planejo cursar ainda o último ano do médio nos eua em um intercâmbio aprimorando assim meu inglês, que já eh considerado intermediário para avançado, sendo que nunca fiz um curso de inglês, fui atrás e aprendi por mim mesma; tenho medalhas em esportes, atualmente luto uma arte marcial, adquirindo certificados de excelência, mas na maioria, as medalhas, são de olimpíadas realizadas na escola, tenho medalhas de natação e corridas, mas não são esportes que ainda pratico, são medalhas antigas;… Enfim gostaria de saber se mesmo com isso, com ótima nota no SAT, TOFFEL, o vestibular de medicina, o premed e etc, não conseguiria nem 10% de bolsa em uma faculdade de medicina nos eua? Por mais que não fosse uma de ponta? Minha intenção eh não sair de lá, ir para morar e ficar. Aguardo retorno

    Responder
    • Olá Giovanna,

      Você ainda está muito longe de poder entrar em medicina nos EUA, pois como explicamos no artigo e nos comentários, medicina é como uma “pós graduação”. Você já precisa ter feito uma faculdade primeiro para depois se candidatar a uma vaga em medicina. Para escolas chamadas “graduate school” como medicina, eles olham apenas para sua vida a partir do início da faculdade. Tudo o que você fez antes, suas boas notas, suas medalhas, suas atividades de voluntariado não importam. As notas que eles olham é o GPA da faculdade, não do ensino médio.

      Contudo, a dificuldade para entrar em medicina, no seu caso, não é uma questão de boas notas e currículo expressivo, mas sim o fato de você ser estrangeira. Como temos discutido bastante aqui, inclusive outras pessoas adicionando à essa discussão, é praticamente impossível um estrangeiro ser aceito em medicina nos EUA. Isso principalmente devido à forma como o processo de pagamento da faculdade funciona. Todos os alunos pegam financiamento estudantil pois a faculdade é muito cara e deve ser paga anualmente (não é mensalidade como no Brasil). Como estrangeiros não qualificam para financiamento estudantil (e não existem bolsas para medicina), as faculdades de medicina se negam a ligar com esses estudantes, nem sequer chamando para a entrevista pessoal (que é a última etapa do processo seletivo). Isso ocorre porque seria um desperdício de tempo por parte deles avaliar um aluno que provavelmente não terá condições de pagar o curso à vista caso fosse selecionado. Mesmo alunos que poderiam arcar com os custos acabam sendo prejudicados por essa política. Há também um preconceito grande especificamente em medicina contra estrangeiros (o que não ocorre com outros cursos). Eles não querem que gente de outros países venha absorver a melhor educação em saúde do mundo para depois voltarem para praticar a profissão em seus lugares de origem. É claro que isso é um preconceito sutil e não escancarado (e nem sequer é verdade – a educação nos EUA não é assim tão boa, mas eles acham que é a melhor do mundo…), mas é perceptível quando vemos pessoas nascidas em outros países, mesmo com Green Card e cidadania Americana, sendo rejeitadas em medicina por nenhum motivo aparente (como o caso do Marcos que escreveu um artigo para este site contando a experiência dele tentando entrar em medicina nos EUA).

      O que recomendamos aqui no blog é a pessoa fazer medicina no Brasil. Passar no vestibular de medicina é infinitamente mais fácil do que entrar neste curso nos EUA, mesmo se o que disse antes não fosse um problema ainda maior. Depois de formada no Brasil, aplique para uma residência médica nos EUA. Isso lhe dará um salário confortável o suficiente para viver aqui enquanto faz sua residência e após o término, também lhe dará condições de conseguir trabalho e poder ficar aqui, se é isso o que você deseja.

      Abraços,

      Diego

      Responder
        • Olá Giovanna,

          O custo varia bastante entre as faculdades, mas é uma média de 50 mil dólares por ano (o curso tem duração de 4 anos nos EUA), sem contar manutenção pessoal (moradia, transporte, alimentação, seguro de saúde, etc.).

          Abraços,

          Diego

          Responder
  32. Olá, muito obrigada por esse artigo que vc esclareceu muitas coisas para nós.
    Vim falar sobre minha filha, ela tem 14 anos, muito dedicada e esforçada, já participou de Olímpiadas de Matemática e Ciências, faz inglês desde criança mas o certo seria ela fazer um intercâmbio para a total fluência.
    Ela tem esse desejo de cursar Medicina desde criança e agora me veio com a ideia de estudar nos EUA e morar lá, ela não tem o interesse de voltar(o que já é um sentido bom relacionado a um trecho do seu artigo) estava pensando e estamos bem animadas, ela pode ser uma boa candidata pro futuro? Ela dança mas eu não sei se isso vale como experiências salutares na escolha sobre medicina.
    O premed, no Brasil há cursinhos de preparação ou não? Eu entendi que é melhor ela fazer o college lá mas por exemplo se ela fazer um curso técnico de enfermagem aqui seria relevável em um momento, mesmo se não valesse como o certificado de college para a “pós-graduação”?
    A Educação da cidade que nós moramos é super boa, mesmo sendo em pública( moramos em SCS) e vou falar a ela sobre essas experiências que eles vão levar em conta.O único problema (ou não) seria a “imigração” que ela faria pra estudar nos EUA e o financiamento da faculdade, que seria muito cara.

    Responder
    • Olá Giovanna,

      Infelizmente, um dos maiores obstáculos para brasileiros que querem fazer medicina nos EUA é a questão imigratória. Para uma menina como sua filha, realisticamente, a única alternativa seria ela se casar com um Americano para conseguir um Green Card. Mesmo assim, ela ainda sofreria discriminação simplesmente por ter nascido em outro país (um país “desconhecido” para a maioria dos Americanos). Vale mencionar que esse tipo de discriminação ocorre somente no curso de medicina, por vários motivos que já discutimos ao longo de outras respostas e artigos.

      Muita gente que está passando ou já passou por esse processo como estrangeiro naturalizado tentando entrar em medicina nos EUA passa por aqui para dar uma opinião. Se você der uma lida por aqui (e em 2 outros artigos que temos aqui no site sobre medicina nos EUA) você verá que o processo é quase como uma roleta russa. É um investimento absurdo de tempo, dinheiro, e esforço, para no final das contas acabar sendo alvo de discriminação e não entrando em nenhuma escola. Eu particularmente não conheço um só estrangeiro (nem estou falando só de brasileiros) que tenha feito medicina nos EUA. Quando eu escrevi este artigo, eu sabia como o processo de aplicação funcionava, mas não tinha ideia de que é praticamente impossível para uma pessoa nascida em outro país entrar nesse curso. Foi após começar a receber feedback dos leitores que eu ampliei as minhas pesquisas e descobri como todo esse processo de seleção funciona na prática.

      Pelo que tenho visto, a melhor e mais segura alternativa é fazer medicina no Brasil (muito mais fácil de entrar e quando ela estiver se formando será na idade em que os Americanos estão entrando em medicina, devido ao período do college e premed). É a residência que dará a ela a oportunidade de trabalhar e viver nos EUA por quanto tempo ela quiser. Na residência sim, há uma porcentagem maior de estrangeiros. A residência não é um curso, é um emprego (um estágio profissional, por assim dizer). Isso significa que ela receberia visto de trabalho H1B (que conduz ao Green Card), um salário básico, porém comfortável, e a chance de ser posteriormente contratada por um hospital (extendendo o visto até o ponto em que ela terá o Green Card definitivo e posteriormente cidadania Americana). Esse sim é um plano realista.

      Abraços,

      Diego

      Responder
    • O Diego está correto. Esses detalhes que tornam sua filha uma estudante exemplar não importam nem um pouco para medicina. Aliás, nada do que aconteceu ANTES da pessoa entrar na primeira faculdade (college) importa para medicina, pois é graduate school. Essas coisas que você menciona são olhadas quando o aluno aplica para o college, a primeira faculdade. Quando o aluno vai entrar em medicina ele já é adulto, já tem 24, 25 anos. Já não importa mais se ele ganhou medalha quando tinha 13, ou se tinha boas notas na escola no ensino médio. O que importa são suas notas na faculdade, se ele fez pesquisa científica, onde fez e onde publicou (o gabarito da publicação importa muito), a nota no MCAT (vestibular de medicina), e é claro, a entrevista pessoal, que é onde os imigrantes são “filtrados”. Como o Diego disse, mesmo que ela consiga um Green Card, o sotaque e o fato dela ter nascido em outro país praticamente matam as chances dela de entrar em medicina. Por mais politicamente incorreto que isso seja, é uma realidade nos EUA. Por diversos motivos,incluindo esse e o custo, simplesmente não vale a pena ficar sonhando em fazer medicina nos EUA (em qualquer outro país, ok, mas nos EUA não). A dica do Diego é muito sensata, faça medicina no Brasil mesmo, depois faça a residência nos EUA. É a residência que dará a ela o direito de trabalhar e morar no país. O povo acha que vestibular é o bicho, eles nunca tentaram entrar em medicina nos EUA! rsrsrsrs

      Responder
  33. Olá,
    Meu filho têm 12 anos e quer muito fazer medicina nos EUA. Eu acho que ele planeja ficar por lá.
    Ele têm notas ótimas, a algum tempo passou em um exame para entrar no Colégio Militar, que já aprovou alguns alunos em Harvard. Ele também toca violão e luta judô.
    Será que ele têm chances?
    A faculdade lá têm preços exorbitantes, não é?

    Responder
    • Olá Ester,

      Estrangeiro só pode fazer medicina nos EUA se tiver Green Card ou for naturalizado Americano. Se ele tem alguma dessas condições, é possível planejar o caminho para chegar lá, do contrário, o ideal é fazer medicina no Brasil e depois fazer residência médica nos EUA. É a residência que dará a ele o visto de trabalho e condições de posteriormente poder viver no país. Veja que medicina nos EUA não é como no Brasil, em que o aluno entra após sair do ensino médio. Este curso é uma “pós graduação” por assim dizer, o aluno precisa primeiro ter feito uma faculdade, e em muitos casos, curso pré-médico. A idade média de entrada em medicina nos EUA é 25 anos, então seu filho ainda está muito longe de até mesmo pensar no assunto. As notas dele atuais não seriam nem sequer condideradas em medicina. Notas da escola valem para entrada no college (equivalente à faculdade no Brasil). Medicina não é college, é a chamada “graduate school”. Só são avaliadas as notas e demais aspectos dos alunos após entrarem na faculdade (college).

      Abraços,

      Diego

      Responder
  34. Estou terminando o ensino médio no Brasil e tenho muito interesse em ir fazer faculdade nos Estados Unidos,mas como não tenho 18 anos ainda conseguirei fazer 6 meses de high school,e gostaria de saber como faço para ingressar em um college,tem como me informar?

    Responder
  35. Estou terminando o ensino médio no Brasil e tenho muito interesse em ir fazer faculdade nos Estados Unidos,mas como não tenho 18 anos ainda conseguirei fazer 6 meses de high school,e gostaria de saber como faço para ingressar em um college,tem como me informar?

    Responder
    • Olá Maria Eduarda,

      Esse é um processo por demais complexo e complicado para explicar em uma simples resposta de comentário. Recomendo que você procure no Google como aplicar para colleges nos EUA. É uma série de procedimentos e documentação para preparar e há muitos sites que se dedicam a cobrir esse processo, então não deve ser difícil você encontrar informações. Se você fizer perguntas para alguém, faça já de um ponto de conhecimento da situação. Pergunte para clarificar pontos que você ainda não entendeu 100%. Perguntar logo de cara qual é o processo inteiro não é a melhor opção! Investigue um pouco primeiro, pergunte depois!

      Abraços,

      Diego

      Responder
        • Não, residência a pessoa já um adulto formado. As atividades extracurriculares já não importam mais. O que é observado são as notas dos STEPs, média das notas na faculdade de medicina, cartas de recomendação de professores da faculdade, médicos com quem o aluno fez estágio, etc., e a entrevista individual. Nesse estágio nada mais importa, só a sua capacidade comprovada, através desses fatores, de ser um bom médico.

          Responder
  36. Olá!
    Me chamo Victor e possuo 24 anos. Este ano, tranquei minha faculdade de engenharia civil, da qual completei 50% do curso, para seguir meu sonho, na real digo que tenho vocação apenas para esta profissão. Bem, após uma lida rápida, percebi que, não é simples fazer residência nos EUA e mais que isso, não tem sentido se, o real interesse for de retorno ao país de origem. As minhas dúvidas são simples:

    1 – Seria burrice eu me empenhar tanto para conseguir tal feito com base na minha idade?

    2 – Sem greencard é praticamente impossível certo? Em razão dos custos de vida.

    3 – Eu tenho como conseguir cidadania europeia através de parentesco familiar. É mais fácil eu ir estudar na Europa e provavelmente, residirei lá. Gostaria de saber algum site que me explique o que seria preciso ou se é semelhante ao americano.

    4 – Realmente é viável se preparar e se organizar, levando em consideração tudo e todos os aspectos? Afirmando que me saia bem em todos os testes. É possível conseguir a vaga?Sei que a xenofobia é constante lá fora.

    Grato.

    Responder
    • Olá Victor,

      Me desculpe se eu soar como um “tio véio” aqui! Trancar a faculdade é uma péssima decisão, pra começo de conversa. Quando a gente é jovem, a gente acredita que o tempo tá passando rápido, que se não fizermos as coisas nesse exato momento, logo, logo, vamos estar velhos demais, a oportunidade vai passar, etc. Essa impulsividade na verdade só atrapalha. No tempo que trabalhei em seleção acadêmica, vi muita gente “mais velha”, 30, 35, 40, 50 anos cujo maior arrependimento era ter parado a faculdade, ou simplesmente ter escolhido “fazer outra coisa” ao invés de um curso superior. Em segundo lugar, sem curso superior, as chances de você conseguir um Green Card são praticamente zero, a única outra alternativa seria você se casar com uma Americana, mas relacionamento você sabe como é… nem sempre dá certo e o casamento tem que durar um certo tempo para que você receba o Green Card definitivo. Mesmo com Green Card, contudo, entramos no terceiro ponto, a vida de uma pessoa sem curso superior nos EUA é miserável! Foi-se o tempo que imigrantes vinham pra cá e faziam trabalhos subalternos, tendo dinheiro suficiente para sobreviver bem aqui e ainda enviar dinheiro pros parentes em casa. O custo de vida é altíssimo e os salários não estão acompanhado.Você está seguindo as eleições aqui? Está vendo que o crescimento da pobreza é uma das maiores preocupações políticas? Essas pessoas “pobres” aqui são os que não têm curso superior. Os empregos que elas conseguem não pagam as contas.

      Você tem chance de ser um engenheiro civil, cara! Engenheiros, de qualquer especialidade, são super procurados aqui e tem seus vistos de trabalho financiados pelas empresas que estão famintas por esse tipo de profissional. O mesmo vale para a Europa.

      Pense bem em como você vai realizar esse seu “sonho”… Você prefere ser impulsivo e vir pra cá agora sem um diploma para ser pobre (e ilegal) para o resto do seu tempo aqui (até o seu sonho perder o brilho e você voltar pro Brasil com o rabo entre as pernas) ou você prefere aplicar mais 2 ou 3 anos para terminar seu curso no Brasil, e vir pra cá como engenheiro, sendo contratado com salário de classe média alta por uma empresa que vai te levar a obter o Green Card, e futuramente cidadania Americana?

      Também não acho que você deve ir para a Europa sem diploma. O processo de cidadania Européia demora anos. Termine a faculdade. Provavelmente você estará pegando seu passaporte Europeu na mesma época em que estará se formando. Infelizmente, não conheço nenhum site para te dar maiores informações, mas com certeza você vai achar muita coisa no Google.

      Abraços,

      Diego

      Responder
  37. olá!
    meu nome é Hellen, tenho apenas 13 anos, irei cursar um ano do ensino médio nos Estados Unidos.
    Sei que sou nova mas acho que já está na hora para começar a me preparar e tenho algumas perguntas
    1- Nao falo ingles fluentemente, irei começar o curso ano que vem, mesmo assim nao vou aprender o necessario para realizar o TOEFEL, o que você recomendaria?
    2-Para entrar no College precisa ter um currículo invejável ou so na med school?
    3- Se nao for possível fazer a college nao seria possivel ir direto para uma universidade de medicina?
    4- sou muito nova para me preocupar com isso?
    5- o comitê iria preferir que eu fizesse o intercambio nos EUA ou em outras partes do mundo?

    Responder
  38. estava lendo os comentarios e vi que você estava aconselhando à fazer primeiro a faculdade de medicina no brasil.
    então, você acha praticamente impossível entrar na med school mesmo tendo feito 1 ano do ensino médio lá, o college, e tendo diversos trabalhos voluntários desde a adolescencia?

    Responder
    • Olá Hellen,

      Se você leu os comentários anteriores, você deve ter percebido que o principal obstáculo para entrar em medicina nos EUA não é o histório educacional do aluno, mas o status imigratório. Somente residentes (portadores de Green Card) e cidadãos Americanos podem se candidatar a uma vaga em medicina. Estrangeiros não podem nem participar do processo seletivo. Além disso, aos 13 anos, você está muito, muito longe de até mesmo pensar em medicina. Medicina, como você já viu aqui, é “graduate school”, você precisa já ter feito uma faculdade antes (college é a mesma coisa que a faculdade no Brasil, não tem diferença nenhuma). Ao aplicar para a graduate school, seja medicina, direito, farmácia, psicologia, tudo o que importa é o que você fez a partir do momento em que entrou na faculdade. Sua vida no ensino médio não importa, nem há espaço para esse tipo de informação no processo de aplicação. Mas mesmo se você fizer college nos EUA, você ainda fica “devendo” os documentos imigratórios para poder aplicar.

      Abraços,

      Diego

      Responder
  39. Muito bom o artigo!! Tenho 18 anos e sai do ensino médio a um ano… Tenho uma vontade imensa de fazer medicina, e no exterior ainda mais. Gostaria de saber a sua opniao e sugestao para o meu caso, faço trabalhos voluntarios a um tempo e vou começar agora no Hospital, além das matérias para ter o mínimo do perfil no que mais deveria investir agora?? Obrigada!

    Responder
    • Olá Mariana,

      Você tem que especificar um pouco mais esse seu sonho! Cada região (ou país) tem suas próprias regras e graus de dificuldade. Você precisa definir o que quer. Fazer medicina “no exterior” pode ser no Uruguai ou na Argentina, onde costumam entrar os que não passam no vestibular no Brasil ou pode significar fazer o curso na Europa, onde o sistema de entrada é igual ao do Brasil (vestibular) mas significativamente mais difícil. Nos EUA é impossível, a não ser que você tenha Green Card ou cidadania. Sabendo disso, você precisa estudar mais sobre como entrar em medicina em cada país em que está interessada e começar a trabalhar em direção a sua aplicação de entrada. Eu só entendo do sistema Americano, não posso te ajudar com outros países.

      Abraços,

      Diego

      Responder
  40. Prezados bom dia.
    Um aluno que já faz High School nos EUA com GP de 5.3 só usa essa nota para entrada no College?
    Você estando em um High School americano tendo iniciado o 10 grade ainda tem tempo para construir esse caminho?
    Alguém teria indicação de pessoas que possam ajudar nesse construção de um caminho para entrada em Medicina em uma universidade nos EUA?
    Fora EUA quais outros lugares e universidades vcs poderiam indicar ?

    Responder
    • Olá Claudia,

      Não existe GPA 5.3. O máximo do GPA é 4.0. Se a escola em que ele estudou calcula GPA de forma diferente, isso precisa ser recalculado de acordo como o padrão usado pelos colleges. GPA é só uma parte da aplicação para o college. Ainda é necessário prestar o SAT (equivalente ao ENEM no Brasil), ter currículo de atividades extracurriculares, sociais, esportes, artes, caridade, etc. Também é necessário ter cartas de recomendação de ex-professores e/ou pessoas de destaque (políticos, empresários, etc.) enviadas pela pessoa direto para cada college em que o aluno está aplicando.

      Medicina não é um curso disponível para alunos estrangeiros sem Green Card ou cidadania Americana. Você não mencionou se esse aluno tem residência Americana. Caso positivo, eu recomendo que você faça bastante pesquisa principalmente em foruns como o StudentDoctor.net pra ir aprendendo como tudo isso funciona. Não é em uma resposta em um blog que você conseguirá obter a complexidade de conhecimento que você precisa nesse caso.

      Quanto a outras faculdades e outros lugares, tudo depende do que o aluno quer alcançar na vida. Qual o objetivo? Por que fazer faculdade no exterior? O que ele quer fazer com o diploma dele? Onde ele quer residir depois que terminar a faculdade? Essas respostas determinam quais as instituiçãoes e quais países seriam adequados pra ele. Não é algo que você se beneficia de dicas aleatórias.

      Abraços,

      Diego

      Responder
  41. Estou planejando fazer biologia nos eua,porém tem como objetivo fazer medicina.Na universidade que farei biologia tem o curso PRE med,nesse curso terei que pagar aparte?

    Responder
    • Olá Lara,

      Pre med não é um curso, é um conjunto de matérias. Algumas faculdades oferecem premed como um conjunto organizado dessas matérias pra alunos que não tiveram oportunidade de cursá-las durante o college, o que não é o seu caso, pois sabendo que você quer fazer medicina, você pode fazer todas essas matérias já no college. Não existe “faculdade de biologia” nos EUA, pelo menos não da forma como existe no Brasil. Nos EUA existe somente o college, e você entra no college, você não entra no college de biologia, ou de matemática, ou de economia, ou de psicologia, etc… Você é aceita no college e a partir daí você mesma organiza como vai fazer as matérias ao longo dos anos para no final se graduar como “major” em biologia. Então você pode fazer todo o premed em qualquer universidade.

      Abraços,

      Diego

      Responder
  42. Oi, tenho 13 anos de idade eu procuro como saber coisas de medicina, e como entrar na Universidade Harvard. Bom e parece que eu já achei um site, e esse site é esse que eu encontrei. VALEU PELAS DICAS, MUITO OBRIGADO……….

    Responder
  43. Acho que valeria a pena aos jovens interessados em cursar medicina nos EUA darem uma pesquisada sobre se alistar nos EUA. Morei lá por um tempo e conheci um rapaz que se alistou e hoje cursa medicina pelo Exército (Army). Obviamente não sei todos os detalhes, mas acredito que seja fácil de conseguir na internet. Acho que vale a pena salientar que se tiver uma guerra vc não terá escolha de pedir baixa depois, e além de tudo vc irá na linha de frente por ser estrangeiro (assim me disseram). Mas tem os pontos positivos também, como ganhar a cidadania(sim, a cidadania) em menos de seis meses e não pagar pela faculdade (até onde eu sei).

    Como eu disse, não fui a fundo nesse assunto com o conhecido que fez isso, e mt menos tenho contato com ele pra perguntar, mas espero ter ajudado a pelo menos dar uma esperança a mais aos interessados no curso. Aconselho aos interessados pesquisarem mais mt mais. Sei que existem outros países que se pode fazer medicina, e ainda outros cursos mais acessíveis nos EUA, como o de enfermagem; pode parecer ridículo eu dizer isso pra alguém que tem um sonho, mas falo mais pq mt gente só quer sair do Brasil e acha que o curso de medicina vai ser o mais seguro para permanecer em outro país…

    Acho que tudo é possível, principalmente pra quem é jovem!!!

    Responder
  44. Olá Diego,
    Sou formada em Engenharia de Produção e Pós Graduada em Engenharia de Segurança do Trabalho no Brasil, mas tenho desde de criança o sonho de ser Médica Veterinária. Eu não cursei o curso de Veterinária porque durante meu período estudantil as atividades e oportunidades que apareceram para mim, me levaram para a area da engenharia. Então, primeiramente gostaria de saber se essas informaçoes de seleção para a faculdade de medicina são as mesmas para medicina veterinaria?
    Para a construção do curriculo, as missoes, voluntariados, rotinas hospitalares em humanas tambem servem para a medicina veterinaria ou não, preciso tambem fazer missoes, voluntariados e participar de rotinas hospitalares específicas da area?
    E a ultima pergunta é; se eu validar meu diploma de engenharia terei as materias de física e química, faltaria as de biologia, isso já conta para as materias do pre med?

    Responder
    • Olá Janda,

      O processo de medicina veterinária é muito parecido com o de medicina sim. Você precisa fazer voluntariado e trabalhar profissionalmente na área, não necessariamente no mesmo nível que medicina (como indo para missões em países pobres como as pessoas fazem). O propósito é demonstrar que você está realmente interessada na profissão e tem experiência suficiente para “ter certeza” de que é isso o que você quer da vida. Você pode aproveitar suas matérias de engenharia sim.

      Abraços,

      Diego

      Responder
  45. Boa noite, queria saber se é possível estudar medicina em Harvard, mesmo sendo brasileira, para morar lá.(seria bolsista)
    Exemplo:
    Na entrevista cito que quero fazer medicina e quando terminar voltar para o Brasil por alguns anos, somente até para me estabilizar e depois voltar para EUA.
    Nesse caso, seria possível entrar em Harvard com essa intenção?

    Responder
    • Olá Elane,

      Não existem bolsas em medicina, nem mesmo para os Americanos. Além disso, estrangeiros sem Green Card não podem sequer participar do processo seletivo, não chegando na entrevista (que é a última etapa do processo).

      Abraços,

      Diego

      Responder
      • Obrigado Diego.É realmente uma pena que não aceitem estrangeiros, seria vários sonhos realizados. Mas os EUA não é o único lugar onde a medicina, então é apenas buscar. Mas uma vez obrigado pela ajuda.

        Responder
        • Olá Elane,

          Acho que sua perspectiva sobre esse assunto pode estar deslocada. O melhor lugar para fazer medicina sendo Brasileiro é no Brasil mesmo, onde você pode fazer o curso de graça em uma universidade federal. Se o motivo pelo qual você cogitou fazer nos EUA foi a dificuldade de entrar em uma universidade federal no Brasil, saiba que nos EUA é infinitamente mais difícil entrar em medicina, isso para os próprios Americanos. Se a dificuldade do vestibular é o que te assusta, outros países da America Latina como Uruguai e Argentina podem ser a solução. Europa também é mais difícil de entrar do que no Brasil. Se a dificuldade do vestibular não é problema para você, então não vejo motivos para não fazer medicina no Brasil.

          Abraços,

          Diego

          Responder
  46. olá,
    Estou no 8º ano do ensino fundamental.Já estou começando a pensar no meu futuro!Ando pensando em fazer neurociência e estava conversando com meu professo de ciências e ele me perguntou se eu não gostaria de fazer medicina, pois tenho um perfil escolar exelente. Então comecei a pesquisar sobre estudar no exterior , pois penso que me adéquo melhor a cultura americana, em fator da minha paixão em novas culturas

    Gostaria de saber o que devo fazer desde já para cursar medicina nos Estados Unidos?

    Responder
    • Olá Luiza,

      Não é possível estudar medicina nos EUA sem ser Americana ou ter Green Card. O que é indicado aqui no site é que quem quer morar nos EUA e praticar a medicina deve fazer o curso no Brasil mesmo e depois vir fazer residência aqui (que é o procedimento para validar o diploma). Contudo, medicina não conduz à neurociência. Ao contrário do que muita gente pensa, neurologia e neurociência não estão de nenhuma forma conectadas. Neurociência é dividida em duas áreas, a área biológica, que estuda o cérebro no nível celular e molecular e envolve trabalho em laboratório. Cursos que podem levar à neurociência são biologia, bioquímica, engenharia biomédica, e engenharia quimica. Há também dentro da área, a divisão biotecnológica que estuda e desenvolve equipamentos médicos de imagem (como ressonância magnética, por exemplo) ou tecnologia relacionada à neurologia e cérebro (chips implatados no corpo humano, por exemplo). Cursos que levam a essa área são engenharia mecânica e engenharia biomédica. A outra área da neurociência, que é a que a maioria das pessoas associa com o assunto, é na realidade psicologia cognitiva. Essa área só é realmente acessível no nível de doutorado. Qualquer formação inferior não lhe permitiria ter qualquer destaque ou mesmo trabalhar na área. Psicologia naturalmente é um precursos para entrar em cursos de doutorado na área (lembrando que nos EUA não se costuma fazer mestrado antes do doutorado), mas cursos como biologia, bioquímica, química, ou uma engenharia relacionada também não é incomum de ver nesses profissionais.

      Acredito ser importante estudar um pouco sobre a neurociência para ver bem o que é que te atrai nessa área (biologia do cérebro, biotecnologia de equipamentos de estudo e melhoria do corpo humano, ou o estudo das “ideias relacionadas ao funcionamento da mente”). Isso pode definir com mais clareza o que é que você realmente quer e pode te dar uma direção mais certeira.

      Todos os outros cursos podem ser feitos nos EUA (faculdade, mestrado e doutorado). É só medicina mesmo que não dá.

      Abraços,

      Diego

      Responder
  47. Olá.. ótimo artigo, esclareceu muitas dúvidas.. eu gostaria de saber se para cursar ODONTOLOGIA nos eua é o mesmo processo?? Eu sei que precisa do undergraduate tbm e depois apply pra ver se consegue entrar na dental school. As minhas dúvidas são: odontologia é TÃO difícil quanto medicina ou o processo é mais simples??? Existe bolsa pra alunos estrangeiros cursarem odontologia nos eua??? (Fazer algum esporte, alguma prova, etc) e você sabe algo em relação a algum programa tipo FIES? Que a faculdade “paga” seu curso e quando vc se forma vc devolve o dinheiro?? Isso existe pra estrangeiros? Se possível responder no meu email.. Muito obrigada

    Responder
    • Olá Bruna,

      Odonto é o mesmo esquema que medicina, quase tão difícil quanto e extremamente antipática com estrangeiros. NÃO EXISTEM BOLSAS em odonto! Nenhum tipo de bolsa, nem para Americano e muito menos para estrangeiro. CAPES no Brasil é um programa que paga para alunos fazerem mestrado e doutorado no exterior como incentivo à ciência. Odonto é um curso profissionalizante, não é científico. Nenhuma instituição paga para você se formar profissional e fazer dinheiro para si mesmo (ao contrário de programas de bolsas que pagam para você e, em troca, você devolve em forma de trabalho acadêmico quando volta para o Brasil). Entende a lógica? Isso não existe para cursos como odonto. O equivalente ao FIES nos EUA, obviamente, só está disponível para Americanos e portadores de Green Card, não para estrangeiros.

      Fazer esporte e coisas do tipo é útil somente na undergrad school (college). Odonto é graduate school, período integral. Os alunos não praticam esporte pela escola. Espera-se que o aluno de odonto dedique **TODO** o seu tempo para o curso, ao contrário do college onde os alunos dividem o tempo em aulas com esportes, atividades extra-curriculares, voluntariado, estágios, etc. Lembre-se: odonto e medicina nos EUA é escola profissionalizante, a dinâmica é completamente diferente de faculdade.

      Se você não tem recursos para arcar com as despesas do curso, mais cursos de vida nos EUA, é praticamente impossível fazer odonto.

      Mas a questão não é bem essa… odonto no Brasil é um curso acessível direito do ensino médio. Tudo o que você precisa fazer é um teste (ENEM, vestibular). Por que você desejaria ter que passar pelo tormento do processo seletivo Americano, ter que fazer outra faculdade antes, e ainda gastar mais de 1 milhão de Reais no total para se formar em um curso que você pode fazer com a mesma qualidade no Brasil (e sem pagar nada)?

      Abraços,

      Diego

      Responder
  48. Diego, minha filha está com um dilema enorme. 1. Não sabe se estuda medicina aqui no Brasil e faz residência ou uma pós graduação nos USA ou 2. Termina o ensino médio e já cursa medicina nos USA mesmo. Caso você pergunte, minha filha tem cidadania americana. Mas ela pensa em excrescer a profissão tanto no Brasil quanto nos USA porque ainda não sabe onde quer morar, pois tem ainda somente 16 anos. Ela diz que quer fazer a residência e pós lá, assim poderá exercer aqui porque a toda a família se encontra no Brasil. Mas creio eu, que ela tera uma serie de dificuldades caso ela decida fazer faculdade por aqui, pois li que ela terá que passar por uma serie de exames. Isso não quer dizer que ela não tera que passar por eles la tambem, mas indo agora, ficaria mais familiarizada com a lingua. Outra coisa, ela também, não ficaria sozinha, eu iria com ela pois tenho o green card. Então Diego, que caminho voce orientaria nos seguirmos? Desculpa. Não sei se consegui ser clara.

    Responder
    • Olá Gislaine,

      Há alguns pontos para vocês considerarem:

      – Medicina nos EUA é um curso de pós-graduação (“graduate school”). Portanto, sua filha não pode entrar em medicina nos EUA após o ensino médio. Ela precisa fazer uma faculdade primeiro (qualquer uma), o que é chamado nos EUA de “undergraduate school”, ou “sub-graduação”. Essa faculdade pode ser qualquer uma feita no Brasil (exceto medicina em si).

      – Mesmo sendo Americana e podendo pegar financiamento estudantil, a situação do débito estudantil nos EUA está fora de controle (o que está gerando uma crise absurda nessa área). Muitos alunos que têm oportunidade de cursar medicina no exterior (Europa, Caribe, e America do Sul) estão fazendo isso para evitar se endividarem de forma que terão muita dificuldade de pagar esses empréstimos depois de formados. A faculdade de medicina é a mais cara, gerando um débito médio de mais de 250.000 dólares no final do curso. Se sua filha tem condições de estudar em uma universidade federal no Brasil e se formar sem dívida alguma, essa, sem dúvida, é a melhor opção.

      – Para praticar medicina nos EUA, um aluno formado em outro país deve fazer residência nos EUA. Isso mesmo que esse aluno já seja médico com décadas de experiência e já tenha feito residência em seu próprio país. Sendo assim, o que sempre indico para meus leitores é que façam medicina no Brasil e depois venham fazer residência nos EUA. A residência nos EUA é aplicada no Brasil, mas o contrário não ocorre, ou seja, se ela fizer residência no Brasil, terá que fazer outra nos EUA, mas se fizer nos EUA, não precisará fazer de novo para praticar a profissão no Brasil.

      – O vestibular/ENEM no Brasil é uma benção. O processo de seleção de medicina nos EUA é completamente insano e exige anos de preparação. Não é uma questão de fazer testes (de teste, tem apenas 1). O problema é a construção do currículo pré-medicina que exige experiência profissional, voluntariado extensivo (muitos alunos chegam a ir para a Africa e outros lugares carentes junto com a ONU ou Médico Sem Fronteiras para “apimentar” o currículo). Isso sem contar as matérias chamadas “premed” que devem ser cursadas com notas absolutamente perfeitas (um mero 8.5 já machuca o aluno) antes da aplicação para medicina. Isso tudo sem contar o fato de que a decisão de entrada em medicina é completamente subjetiva. Não importa quão boa ela seja, quão boas sejam suas notas, currículo, etc., cada comitê de cada universidade se reserva o direito de selecionar pessoalmente cada aluno de medicina, o que abre precedente para muita discriminação (leia o artigo do Marcos aqui no site sobre sua experiência como Brasileiro naturalizado Americano tentando entrar em medicina nos EUA).

      Minha opinião é que ela deve fazer o curso no Brasil. Será muito mais rápido, já que no Brasil medicina está disponível logo após o ensino médio. Ela se formará sem débito e poderá posteriormente aplicar para uma residência nos EUA.

      Desejo boa sorte para sua filha!

      Diego

      Responder
  49. Quanto a residencia medica, após terminar a faculdade de medicina aqui, posso me candidatar a residencia la? É necessario o green card para isso tambem? Como isso funciona? Visando que nao terei tempo de morar la tempo suficiente para conseguir um green card antes de entrar na residência

    Responder
    • Olá Maria,

      Residência é um emprego, não é um curso. Se você for aceita em um programa de residência, o hospital escola precisaria solicitar um visto de trabalho para você (isso ocorre com frequência em programas de residência). Não é necessário Green Card.

      Abraços,

      Diego

      Responder
  50. Meu filho estuda Direito em uma faculdade privada em Porto Alegre e tem interesse em fazer pós graduação nos Estados Unidos ou na Europa. 3ele está no 3 ano e é cidadão italiano. Alguma sugestão?

    Responder
    • Olá Marta,

      Se ele tem cidadania Européia, eu recomendaria a Europa pela facilidade de receber dinheiro público e bolsas para estudar. Infelizmente não conheço a fundo o ensino na Europa para te indicar algo mais pontual, mas conheço algumas pessoas que fizeram pós em direito na excelente Universidade de Bolonha (a mais antiga da Europa) e recomendam.

      Abraços,

      Diego

      Responder
  51. Minha filha tem 16 anos e está terminando o ensino médio em um colégio de ponta em Brasília. Seu histórico escolar é excelente, com notas altas.
    Quer fazer medicina e morar nos EUA.
    Se ela fizer o College lá ela tem chances de conseguir cursar medicina?
    Ou é melhor ela fazer a faculdade aqui no Brasil e tentar a residência lá?

    Responder
    • Olá Flávia,

      O problema especificamente com medicina e estrangeiros é que esse curso, nos EUA, não está disponível para estrangeiros que não tenham Green Card ou não sejam naturalizados Americanos. Em casos raros, alunos estrangeiros são aceitos com a condição de que paguem o curso todo à vista (em torno de 200 mil dólares), mas em geral alunos estrangeiros não conseguem nem fazer a aplicação eletrônica que dá início ao processo. O ideal é fazer no Brasil mesmo, até porque ela termininaria em apenas 6 anos saindo do ensino médio, ao invés de ter que fazer um curso superior, passar mais alguns anos preparando o currículo para medicina nos EUA (em geral os alunos passam 2 anos nessa preparação) e depois mais 4 de medicina. O processo de entrada na residência, apesar de difícil, é mais fácil do que entrar na faculdade em si e lembre-se de que residência não é um curso, é um emprego. Ela teria salário e visto de trabalho, podendo após o término da residência, ter a oportunidade de receber uma oferta permanente de trabalho e poder ficar no país (se é essa a vontade dela).

      Abraços,

      Diego

      Responder
  52. Olá Diego. Tenho 20 anos e daqui uns dias estarei indo para os EUA para entrar em processo de legalização…não sei quanto tempo demorará a pagar o green card mas o meu social security deve sair de imediato, com esse documento pode entrar na faculdade enquanto aguardk pegar o green card? Não fiz favuldade ainda mas já pretendia morar no nos eua (meu plano era fazer medicina aqui no brasil e depois tentar fazer a prova de residência) mas por varios Motivos vi que seria mais “facil” fazer a faculdade nos eua do que aplicar para a residência depois… além da minha dúvida sobre o social security, eu pretendja aplicar primeiro para uma community college e depois fazer uma college/university para graduação em biologia ou fazer o pre med…é possivel esse processo por meio de uma community college? Outra pergunta, vi a matéria sobre um brasileiro que fez a graduação em ciências biologicas no Brasil e depois fez o pre med nos EUA e mesmo tendo feito tantos serviços voluntários não foi chamado para fazer medicina no país…minha duvida É: mesmo tendo feito o college ai nos eua, tudo certinho eu posso não ser chamada por ser brasileira? Mesmo ja Tendo meu green card/cidadania. E é possivel estudar e trabalhar? Mesmo com a ajuda dos meus pais para arcar com o custo eu pretendo ajudar eles. Muito obrigada Diego.

    Responder
    • Olá Ana,

      Quanto a estudar, você pode estudar de qualquer jeito, com ou sem Green Card. Até mesmo imigrantes ilegais estudam nos EUA. Nesse caso, eles precisam pagar, já que não qualificam para financiamento estudantil. Você
      poderá qualificar para financiamento assim que pegar seu Green Card. A entrada em medicina é complicada… fica difícil falar em probabilidades de conseguir entrar ou não. Conheço muito Americano que fez tudo direitinho e nunca conseguiu entrar. Quando o processo seletivo é subjetivo, tudo fica à critério dessas pessoas que escolhem os alunos, não é um processo objetivo como vestibular em que você pode estimar suas chances de entrar em determinada faculdade com base nas notas de seus simulados.

      É importante fazer uma faculdade em si, não só premed. Hoje em dia, é impossível entrar em medicina só com premed e se você fizer uma faculdade como biologia ou bioquímica, não precisará perder tempo fazendo premed. Lembre-se que premed é praquele aluno que fez um curso como psicologia, ou geografia. Esse aluno não tem todos os pré-requisitos para entrar em medicina, então ele faz premed só para completar os créditos que ele precisa.

      É possível e mais recomendado começar o processo em um community college, você ecomoniza bastante dinheiro (mesmo que esteja estudando com financiamento). Nesse caso, você também não precisa se preocupar com SAT, ACT e outros testes que são exigidos de quem entra na faculdade logo após o ensino médio. Outro ponto positivo de fazer community college é adquirir experiência de pesquisa científica que depois será crucial tanto para transferir para uma faculdade boa quanto para entrar em medicina. Por que eu digo isso se pesquisa científica é o forte de universidades e não de community colleges? Grandes universidades de pesquisa têm seus alunos de mestrado e doutorado trabalhando nos laboratórios, fazendo pesquisa “de verdade”. Os alunos de graduação são relegados à “trabalho escravo” e raramente conseguem publicar qualquer coisa. Em um bom community college que tenha um programa de estímulo à ciência (procure por programas STEM), você tem um monte de PhDs (os professores) sem alunos de pós-graduação para ensinar. Então esses professores pegam os alunos de graduação que estão motivados para aprender e lhes dão acesso ao laboratório inteiro. Ou seja, você tem oportunidade de fazer coisas que só um aluno de mestrado e doutorado faz em um community college e sendo assim, tem oportunidade de publicar pesquisa científica (importante para entrar em medicina).

      Para medicina, trabalho voluntário e profissional na área da saúde é super importante. Muitos fazem EMT (ambulância), auxiliar de enfermagem, scribe (os profissionais que tomam notas junto dos médicos). Isso precisa ser feito já desde que você comece o community college ou faculdade. Quando mais anos de experiência você tiver (e experiência variada) melhor. Você pode trabalhar (não integral) enquanto faz faculdade, mas não pode trabalhar enquanto faz medicina. A faculdade de medicina ninguém paga antecipadamente, então não é algo com que você deva se preocupar. Como você terá seu Green Card, você qualificará para financiamento e fará um empréstimo como todos os alunos fazem). Os valores em medicina podem chegar a 60 mil dólares por ano, é insanidade querer pagar o curso enquanto você está na escola.

      Abraços,

      Diego

      Responder
  53. Ola, se eu me formar no brasil (no ensino medio) em uma escola americana (que recebo o diploma brasileiro e americano de conclusão do ensino médio) como faço para aplicar uma faculdade de medicina???

    Responder
    • Oá Gabriela,

      Medicina nos EUA é “pós-graduação”, você precisa de um diploma universitário primeiro e completar todos os requisitos chamados de pré-medicina (premed). Não é possível ir do ensino médio para medicina, não importa onde você cursou o ensino médio ou se seu diploma é Americano ou Brasileiro. O ensino médio não tem nenhum impacto na entrada em medicina nos EUA. Dê uma lida melhor por aqui para entender melhor esse processo.

      Abraços,

      Diego

      Responder
  54. Olá, meu nome é Joyce, eu fiz meu ensino médio no instituto federal de Mato Grosso na área de Química que duraram 4 anos, nesse tempo meu coeficiente de rendimento foi 8,5, e comecei o curso de Química na UFMT, porém, eu tranquei e optei por medicina mas pretendo fazer no exterior, como atividades extracurriculares eu tenho prêmios em 2º e 3º lugar no Comgresso Brasileiro de Química, e perante o ingresso na faculdade e viagens para o congresso fiz cursos pertinentes a área como uma formação complementar como química forense e outros, tenho também premiações mas 3 colocações na modalidade de natação nos jogos os quais participei representando o instituto, mas ainda não tenho nota de proeficiencia em inglês pois, estou me preparando para a prova. Tenho cartas de recomendações, mas não sei se me aplico como uma candidata apta a concorrer essas vagas tão concorridas. Mas, pretendo me formar e seguir a vida por lá, sem retorno ao Brasil, aguardo desde já uma resposta que possa me orientar, Att.

    Responder
    • Olá Joyce,

      Medicina nos EUA é um curso de pós-graduação. Sendo assim, você não pode se candidatar a uma vaga sem ter primeiro um curso superior. Contudo, as chances de entrar em uma faculdade de medicina nos EUA tendo feito curso superior no exterior (Brasil, por exemplo) são muito baixas. Mesmo alunos de países como Reino Unido e Alemanha têm muita dificuldade de entrar em medicina nos EUA. O que temos recomendado por aqui é fazer medicina no Brasil e depois aplicar para uma residência nos EUA. Dessa forma, você obtém visto de trabalho e pode morar e trabalhar no país. Veja que simplesmente fazer um curso superior nos EUA (ou em qualquer outro país) não lhe dá o direito de continuar no país depois de formada, nem de trabalhar. Para isso, você precisa de um visto de trabalho.

      No seu caso, eu recomendaria terminar medicina no Brasil mesmo. Suas atividades extra-curriculares são muito boas para college, mas não para medicina. O que conta como “extra-curricular” em medicina é ter trabalhado como enfermeira, em ambulâncias, ter voluntariado na ONU, Cruz Vermelha, Médicos Sem Fronteiras, esse tipo de coisa. Medalhas, esportes, etc., só importa para os aluninhos de 17 anos entrando no college. Depois do college, essas coisas se tornam irrelevantes. É importante que você se informe mais para saber como funciona o sistema educacional Americano, pois essa diferença de alguns cursos serem college (undergraduate school) e outros serem graduate school confunde muito.

      Abraços,

      Diego

      Responder
  55. Olá! Meu nome é Mario. Tenho duas filhas, uma com 6 e outra com 9 anos de idade. Poderia, com todo o respeito de minha parte e com toda a gentileza da sua, me informar um passo-a-passo para montar uma estrutura boa para que minhas filhas possam entrar em um college nos EUA? Desde já agradeço por sua atenção.

    Ps.: Ótimo artigo!

    Responder
    • Olá Mario,

      Tudo depende do nível das universidades. Há muitas escolas boas como as universidades estaduais que aceitam praticamente qualquer aluno com notas razoavelmente boas no ensino médio e SAT. Contudo, se sua intenção é enviar suas filhas para Harvard, Stanford, Yale, ou escolas de alto nível é importante preparar o currículo delas cuidadosamente com atividades extracurriculares, principalmente atividades que as coloquem em posições de liderança, esportes, artes, línguas, e envolvimento com causas humanitárias/voluntariado. A análise que é feita considera somente o ensino médio, mas treinamento em atividades artísticas (como tocar instrumentos musicais), línguas, e esportes precisa começar desde cedo. O ideal é que elas façam ensino médio em uma escola que tenha experiência em enviar alunos para faculdades no exterior. Também é interessante que no ensino médio elas façam 1 ano de intercâmbio no exterior (não precisa ser no país em que elas pretendem cursar faculdade). Qualquer experiência internacional é válida. Mesmo que isso atrase 1 ano no ensino médio, não tem problema, pois o colegial dos Americanos dura 4 anos. É importante também que vocês estejam preparados para aplicar para universidades durante o último ano do ensino médio. Passar 1 ou mais anos entre o final do ensino médio e a entrada na universidade é mal visto nos EUA (a não ser que o aluno vá fazer alguma coisa “muito interessante” nesse período) e pode diminuir muito as chances de ser aceito nas melhores escolas.

      Nesse ponto, não há muito o que vocês possam fazer além de colocar as meninas em aulas de línguas, artes ou esportes (o que elas preferirem) e incentivá-las a sempre manter as melhores notas na escola. Adotar métodos de estudo que os alunos nos EUA usam (como Flashcards, quizzes, e outras técnicas que tornam os alunos Americanos mais eficazes no aprendizado), assim como estruturar e organizar o aprendizado de cada matéria como se faz nas escolas Americanas (procure sobre syllabus), praticar produção de essays e papers (redações amplamente usadas no ensino Americano), se familiarizar com literatura Inglesa e Americana (elas devem procurar ler obras em inglês mesmo para se acostumarem com a língua mesmo que façam aulas de inglês). Isso as preparará não só para entrar em uma boa instituição, mas também para que possam se dar bem dentro da estrutura de ensino nos EUA, que é muito mais organizada e exige que o aluno esteja sempre “por cima” do que está acontecendo em cada matéria.

      Assim que elas entrarem no ensino médio, aí sim, tudo o que elas fizerem será minuciosamente analisado. Para entrar nas escolas top, elas precisam de notas absolutamente perfeitas (um mero 8 pode arruinar as chances de um aluno entrar em Harvard, por exemplo). Mas se elas desenvolverem hábitos de estudo como fazem os Americanos, com alta estruturação e organização, elas tem grandes chances de entrar nas melhores escolas. Durante o ensino médio, elas devem estudar para o SAT (o ENEM Americano). Esse teste pode ser prestado no Brasil. Da mesma forma, o TOEFL, que é o teste de proficiência na língua inglesa, deve ser prestado no máximo 2 anos antes da aplicação e pode ser feito no Brasil.

      Abracos e boa sorte!

      Diego

      Responder
  56. Hi , otima materia , meu filho esta no ultimo ano de undergraduation em bioquimica na Johns Hopkins University se preparando para o MCAT e posso dizer que e extremamente dificil para um estrangeiro (brasileiro no caso) ingressar em medicina nos EUA por inumeras questoes , boa sorte a todos !!!

    Responder
  57. Ola!
    Meu nome e Tania…
    Minha filha tem 16 anos e nascida nos Estados Unidos e fz o High School la.
    No meio do ano q vem ela termina ela esta em duvida qual curso fazer no College ja que tem o sonho e medicina.
    Mas primeiro tem que fazer o College ela mora na regiao de Boston.
    Pensou de se alistar no exercito nao sei se ajudaria.
    Ela toca piano e violao Cello, pensou ate de fz musica no College mas sai totalmente fora do sonho de de fz medicina.
    Voce poderia nos indicar o melhor caminho.
    Aonde ela pode fazer um curso para socorrista de ambulância ,ou ate mesmo de auxiliar enfermagem.

    Muito Obrigado.

    Responder
    • Olá Tânia,

      Eu recomendo que vocês conversem com um counselor especialista em medical school admissions. Ela está na idade certa para começar a se preparar e precisa de uma estratégia muito bem desenhada, pois cada passo que ela der a partir do momento em que ela entrar no college contará a favor ou contra a entrada em medicina. Atividades extra-curriculares, artes, esportes, isso só conta para entrar no college. Em medicina, nada disso importa, então o ideal é ela não perder o foco com coisas irrelevantes, a não ser que ela tenha uma grande paixão por alguma arte ou esporte. Nessa fase, ela precisa se concentrar em ter as melhores notas na high school e tirar a maior nota que ela puder no SAT para que ela garanta entrada em um college de calibre (o que não falta em Boston). Se ela quer fazer medicina, o ideal é que escolha um major no college que ofereça as matérias premed. Os mais populares para alunos pré-medicina são biologia, química, e bioquímica. Ela não deve fazer curso de socorrista, EMT, enfermagem, etc., agora. Ela precisa fazer o college primeiro. Essas atividades os alunos fazem no interim entre o college e a entrada na medical school.

      Entrar no exército não ajuda, só atrapalha! Ela já mora na região mais rica dos EUA educacionalmente. É só ela jogar as cartas dela com estratégia que medicina pode estar no futuro dela.

      Abraços,

      Diego

      Responder
  58. Oi, tenho 18 anos e estou cursando o 3º ano do ensino médio, a ideia de fazer medicina fora do Brasil surgiu a pouco tempo, meu desejo de fazer medicina surgiu desde os meus 14 anos, estava lendo ali sobre o “currículo”, eu danço desde os 4 anos de idade, comecei com ballet clássico, fui ginasta participando de campeonatos e representando minha cidade e agora faço danças populares com participações em varios festivais de dança, sou catequistas de crisma na minha igreja (não sei se vale como trabalho voluntário), estou terminando meu intensivo de inglês agora, não entendi muito bem os 9 cursos premed, queria saber um pouco mais e saber também se tem como fazer ele durante um intercâmbio

    Responder
    • Olá Carla,

      Esse artigo fala sobre fazer medicina nos EUA, que é um processo completamente diferente de outros países. Praticamente todos os países, exceto EUA, tem um esquema de vestibular similar ao Brasil. Nos EUA, contudo, existem 2 fases, o college, que é uma faculdade equivalente à do Brasil e a graduate school, que é a escola profissionalizante. Medicina é “graduate school”, ou seja, você precisa fazer college, ou seja, uma faculdade completa, antes de poder entrar em medicina nos EUA. Dessa forma, você com 18 anos ainda não pode se candidatar a uma vaga em medicina. Também vale mencionar que atividades extra-curriculares como as que você faz são importantes para o college apenas, não para medicina. Extracurriculares para medicina são as atividades relacionadas à medicina em si como trabalhar como auxiliar de enfermagem, ambulância, ajudante de médico, etc. Se você não tem intenção de fazer medicina nos EUA especificamente, dê uma olhada em outros países. O processo é muito mais simples, rápido, em alguns países até mesmo gratuito. A probabilidade de um estrangeiro (sem Green Card ou cidadania) entrar em medicina nos EUA é quase zero. Há uma discussão aqui no site sobre essa dificuldade.

      Abraços,

      Diego

      Responder
  59. Olá Diego, tudo bem?

    Excepcional seu artigo! Parabéns!!!

    Então, minha filha concluiu o Ensino Médio aqui no Brasil mês passado (dez/18). O desejo dela agora é fazer medicina. Ela é cidadã americana (nasceu na FL). Você poderia me informar (ou enviar artigo) sobre como ela poderia entrar num College aí nos EUA?

    Um abraço e obrigado.

    Responder
    • Olá Jonecir,

      Como o artigo explica, medicina nos EUA é um curso de pós-graduação (graduate school). Sua filha com 18 anos ainda não pode entrar em medicina, ela precisa fazer uma faculdade primeiro (chamada de undergraduate school ou college nos EUA). Essa faculdade pode ser feita no Brasil. O ideal é que o curso esteja alinhado com a área biológica para que ela não perca tempo depois preenchendo os pré-requisitos pré-médicos. Ela pode fazer biologia, por exemplo, farmácia ou bioquímica também são bons cursos pré-medicina. Veja que a aplicação para o curso de medicina exige uma série de “elementos”, não só cursos acadêmicos, mas também vasta experiência profissional na área da saúde, volutariado, e de preferência, experiência com pesquisa científica. Há muitos sites nos EUA que se dedicam exclusivamente a esse processo de aplicação para medicina, então eu recomendo que vocês pesquisem mais para que ela comece a montar um plano de ação para se preparar adequadamente. Gosto bastante do site Student Doctor (.net) que inclusive tem um forum de discussão onde o pessoal troca ideias e tiram dúvidas.

      Abraços,

      Diego

      Responder

Deixe um comentário ou pergunta